Exam v1 Block 5

¡Supera tus tareas y exámenes ahora con Quizwiz!

4 Pasteurella sp. Explanation The correct answer is Pasteurella sp. Approximately 75% of cat bite isolates are Pasteurella species. Approximately 50% of dog bite isolates are also Pasteurella species. With that in mind, animal bite wounds are a microbiologic mixture of many possible organisms, and wounds should be cultured to ensure that the antibiotic prescribed covers the organisms isolated. All of the other choices are potential pathogens found in cat and other animal bite wounds, but they are not as common as Pasteurella species. The clinical presentation of these other options mirror infection by Pasteurella sp. but are less likely to occur due to a cat bite. Pseudomonas is a gram-negative organism often associated with a pungent odor. Staphylococcus and Steptococcus are gram-positive organisms commonly found in skin infections. Bacteroides sp. are also gram-negative and are found in normal gut flora

A 10-year-old boy presents after being bitten by his cat on the dorsum of his left hand several hours ago. He was playing with the cat, and she snapped and bit him on the hand. The bite drew blood, and his mother immediately washed the bite out with soap and water, but it has gotten increasingly red and swollen over the past several hours. He is currently afebrile. The dorsum of his left hand reveals a 4 cm oval area of redness that is tender to the touch; there are 4 puncture wounds at the center without pus. You suspect the wound is infected, so a culture is taken. Question What is the most likely isolate in this case? 1 Pseudomonas sp. 2 Staphylococcus sp. 3 Streptococcus sp. 4 Pasteurella sp. 5 Bacteroides sp.

5 Mitral regurgitation Explanation Acute rheumatic fever occurs after a pharyngeal group A streptococcal (GAS) infection. It most commonly occurs in children between 3-15 years. The Jones criteria are used to diagnose the disease. If a patient has a history of GAS infection and 2 major criteria or GAS infection and 1 major and 2 minor criteria, the diagnosis of acute rheumatic fever can be made. Major criteria: Carditis and valvulitis Chorea Erythema marginatum Subcutaneous nodules Minor criteria: Arthralgia Fever Prolonged PR interval Elevated serum markers of acute phase reactants Other infectious diseases may also cause arthritis. Erythema infectiosum is associated with human parvovirus B19. Erythema migrans is associated with Lyme disease. Jaundice is associated with Hepatitis B. Fever is a non-specific finding in a patient with inflammatory arthritis.

A 12-year-old girl presents because her joints are hurting. Her knees and ankles have been painful, warm, and swollen for several weeks now. Symptoms first started in her left knee, then her left ankle became involved, and then her right knee was affected. Her left knee is "almost back to normal now," but the other involved joints are becoming more painful. She is a middle school student and has been unable to go to school for the past 3 days because of the pain. She is taking acetaminophen arthritis formula, but the pain continues to get worse. Her mother did not want to give her non-steroidal anti-inflammatory medications because of her age. Review of systems (ROS) is positive for fever, chills, malaise, mild chest pain when lying down, moderate headaches, and weight loss. ROS is negative for sexually transmitted diseases, trauma to affected joints, tick exposure, and recent travel. Question What component of the physical exam would lead you to suspect acute rheumatic fever as the cause of her joint pain? 1 Erythema infectiosum 2 Erythema migrans 3 Fever above 100°F 4 Jaundice 5 Mitral regurgitation

2 Look for finger malposition when the fingers are flexed into the palm. With a fractured fifth metacarpal, there is an increased incidence of malrotation of the distal digit due to the potential loss of metacarpal height and lack of ligamentous support of the metacarpal head by the intermetacarpal ligaments. Therefore, rotation must be checked closely to ensure proper position of the distal fracture segment in relation to the proximal fracture fragment for functional healing to take place. This is best accomplished by asking the patient to make a partial fist (gently) and watching for finger malposition (normally all the digits of a closed fist point towards a single spot on the scaphoid) or "cross-over." Be sure to compare the injured hand to the uninjured one as some "normal cross-over" can be seen with the fifth metacarpal. Looking at the fingernails in the extended position may not show rotation as easily as in the flexed position. Although there may be "folds of skin" over the metacarpal head, this wouldn't show you the malrotation if there is any. A fracture of the fifth metacarpal will not stop motion of the PIP or DIP joints (although it may cause pain at the fracture site). Edema is a given accompaniment with a fracture, but it won't give you an indication of a fracture rotation.

A 16-year-old boy presents following the striking of a wooden door with a closed fist an hour ago when he was angry at his mother. He is neurovascularly intact, and the skin is closed. There is an obvious deformity with a loss of small finger metacarpal knuckle. Radiographs reveal an oblique mid-shaft fracture of the fifth metacarpal with a palmar angulation of 45°. Question What physical exam technique must be performed to check for rotation of the fracture? 1 Look at the fingernails while fingers are extended for position in relationship with each other. 2 Look for finger malposition when the fingers are flexed into the palm. 3 Look for folds in the skin over the metacarpal head. 4 Check for motion of the PIP and DIP joints. 5 Look for palmar edema of the affected digit metacarpal.

2 Acne vulgaris Explanation The patient has the presence of comedones, which are characteristic of acne vulgaris. Acne vulgaris is commonly seen in the adolescent male population. While the lesions associated with rosacea resemble acne, it also presents with facial flushing and erythema, eye irritation, and occasionally an enlarged nose. It more commonly presents in middle-aged women. Contact dermatitis usually presents with erythematous pruritic lesions. Erythema multiforme presents with target lesions, usually on the distal extremities. Impetigo presents with honey-colored crusted lesions. It is more common in children.

A 16-year-old boy presents to his primary care physician with skin lesions. The lesions are scattered over his forehead, nose, and chin. He denies facial flushing and pruritus. The following presentation is seen on examination. Refer to the image. Question What is the most likely diagnosis? 1 Rosacea 2 Acne vulgaris 3 Contact dermatitis 4 Erythema multiforme 5 Impetigo

4 Vitamin D deficiency Explanation In cases of vitamin D deficiency, children can present with an inability to walk unsupported due to muscle weakness and lower limb skeletal deformities such as genu varum (bow legs) and genu valgum (knock knees). On examination, they may have prominent costochondral junctions (rachitic rosary) and indentation of the lower ribs at their diaphragmatic attachment (Harrison's groove). Causes include inadequate dietary intake and inadequate exposure to sunlight. They should be encouraged to be exposed to ultraviolet irradiation of the skin as well as increase their dietary sources of vitamin D, including fortified milk, fish liver oils, butter, egg yolk, and liver. Vitamin B1 (or thiamine) deficiency results in beriberi, which is characterized by: a bilateral symmetric peripheral neuropathy beginning in the legs Wernicke-Korsakoff syndrome, which is comprised of nystagmus, ophthalmoplegia, ataxia, memory loss, and confabulation congestive heart failure with tachycardia, peripheral edema, and cardiomegaly In cases of riboflavin (or vitamin B2) deficiency, patients present with angular stomatitis and cheilosis. On examination, they are pale, have atrophic glossitis, and the tongue may appear magenta. In cases of vitamin K deficiency, patients present with bleeding tendencies, which include epistaxis, menorrhagia, and hematuria. The prothrombin time (PT) and the activated partial thromboplastin time (aPTT) are usually prolonged. In cases of iron deficiency, patients can present with feeling weak, dizzy, and tired; they may experience syncope. On examination, they have pale conjunctivae and koilonychia.

A 3-year-old boy presents with his mother; she reports that he is unable to walk without support. He is an only child who rarely leaves his tenth-floor apartment home. On examination, he has a rachitic rosary and genu valgum. Question What is the most likely diagnosis? 1 Thiamine deficiency 2 Riboflavin deficiency 3 Vitamin K deficiency 4 Vitamin D deficiency 5 Iron deficiency

3 Noise-induced hearing loss Explanation It is estimated that 12% of American youth between 12 and 19 years of age have noise-induced hearing loss (NIHL). The louder the sound, the shorter the time it takes to cause hearing loss. Continuous exposure to sound >85 dB can cause damage. There may be a genetic predisposition to NIHL. Damage can be caused by a single sound, such as a gunshot or explosion, or continuous exposure to loud noises. Sometimes, a single loud noise can cause a temporary loss called a threshold shift, which produces ringing in the ears after the event. An early symptom of noise-induced hearing loss is trouble understanding speech when there is increased background noise. Symptoms may include distorted or muffled speech, ringing in the ears, or difficulty in hearing in the classroom. Loud noises damage hair cells in the inner ear that send noise impulses to the brain. Otoscopic examination is generally normal, and rarely reveals tympanic membrane disruption or damage to the ossicles. The Weber test, done by holding a tuning fork on the middle of the patient's head, is only useful with asymmetric hearing loss. The sound localizes to the side with a conductive hearing loss or away from the side with a sensorineural hearing loss. When the sound is heard in the middle of the forehead, it indicates either that hearing is normal or there is a symmetric hearing loss. The Rinne test, done by placing a tuning fork over the mastoid and then moving it to the external auditory canal when sound can no longer be heard, demonstrates air conduction greater than bone conduction with normal or sensorineural hearing loss and bone conduction greater than or equal to air conduction with conductive hearing loss. Chronic NIHL usually requires 10 - 15 years of chronic exposure to constant noise, which is more damaging than intermittent noise. It is always sensorineural, equal bilaterally, rarely produces a profound loss, no longer progresses when the noise is removed, and higher frequencies are more commonly affected. The loss is permanent. It is seen in a wide variety of occupations including factory workers, miners, armed forces, aviation workers, railroad workers, and construction workers. Meniere's Disease presents with attacks that may include vertigo, tinnitus, and hearing loss. Affected individuals tend to be sensitive to loud noises. Attacks may last from 20 minutes to 24 hours. Hearing loss is usually unilateral and usually progresses to a permanent sensorineural hearing loss. Typical age of onset is 40 - 60 years. It is rare in children. Both the cochlear and vestibular systems are affected. Otosclerosis may appear as early as 7 - 8 years of age, but it most commonly presents between 15 and 35 years of age. Hearing loss and tinnitus are the presenting symptoms but vary in severity and rate of progression. Dizziness occurs in some. Causes may include a prior history of frequent otitis media leading to ossicular discontinuity, necrosis of the incus, and replacement of the union between the incus and stapes with a fibrous band. Most commonly, otosclerosis produces a conductive hearing loss due to fixation of the footplate of the stapes in the niche of the oval window. However, it may cause sensorineural hearing loss if it involves other areas of the cochlea. Surgical treatment generally consists of stapedectomy in both children and adults or cochlear implant in advanced cases. Fluorides have been used and may slow or halt the progression of the disease. Sodium fluoride, an enzyme inhibitor, reduces osteoclastic bone resorption and may rebuild pseudohaversian bone. Congenital stapes fixation is detected in the initial 10 years of life and presents with a non-progressive hearing loss. Acquired stapes immobility resulting from tympanosclerosis that fills the oval window with tympanosclerotic plaques. It often results from chronic otitis media and is more common than otosclerosis. Paget's Disease and osteogenesis imperfecta result in stapes fixation and present clinically like otosclerosis. Audiometric evaluation shows conductive hearing loss with an absent stapedius reflex.

A 16-year-old boy presents with ringing in his ears. His mother states that, over the past year, he has had the volume on the television and radio turned up very high. He prefers to listen to music with headphones, and he plays the drums in both the school band and a rock band. His mother has observed that he is uncomfortable in crowded rooms with a lot of people talking. His grades in school have also been deteriorating. He has no prior history of ear infections. Examination of the ears demonstrates normally formed auricles. Ear canals are clear. Tympanic membranes appear normal, and they move well with insufflation. A Weber test localizes sound to the middle of the forehead. Rinne test demonstrates air conduction greater than bone conduction bilaterally. Neurological examination demonstrates no focal findings. A screening audiogram demonstdBtes a 30 dB hearing threshold at 4000 Hz. He was referred for formal audiology testing which demonstrated a symmetric hearing loss most pronounced at 4000 Hz. Question The history, physical examination, and audiogram are consistent with what condition? 1 Congenital Stapes Fixation 2 Meniere's Disease 3 Noise-induced hearing loss 4 Otosclerosis 5 Tympanosclerosis

1 Pregnancy Explanation Secondary amenorrhea is the cessation of menstruation for ≥3 consecutive cycles after at least 24 months of regular monthly cycles when a normal ovulatory pattern should have been established. This most commonly results from some disruption of the hypothalamic-pituitary-ovarian axis. Natural causes of secondary amenorrhea include breastfeeding, menopause, and pregnancy. Pregnancy is the most common cause of secondary amenorrhea in adolescents. It should be considered and tested for in all situations. Denial of sexual activity does not preclude the need to rule out possible pregnancy, and it should be explored in this patient. Stress-related conditions are a common form of disruption of the hypothalamic-pituitary-ovarian axis, contributing to functional hypothalamic amenorrhea. By definition, this disorder excludes pathologic disease and is described by a decrease in hypothalamic GnRH secretion. A hypothalamic state is induced, which reduces secretion of gonadotropin-releasing hormone (GnRH), producing levels of luteinizing hormone (LH) and follicle-stimulating hormone (FSH) that are too low to stimulate ovulation. Estrogen is therefore not produced by the follicle that would normally support the proliferative phase of the endometrium. The triad of eating disorder, amenorrhea, and osteoporosis (also known as the female athlete triad) is well recognized among young women competing in certain sports emphasizing leanness. Secondary amenorrhea can be an early signal of anorexia nervosa, occurring before dramatic weight loss is apparent. Failure of normal ovarian function resulting in amenorrhea can also result from autoimmune processes often associated with thyroid dysfunction and diabetes mellitus. Poorly controlled diabetes is associated with anovulation. Patients will have low levels of estradiol and elevated FSH levels from an absence of a normal negative feedback system. Nothing in this patient's history alludes to a possible autoimmune problem. Craniopharyngioma (or pituitary adenoma) are found in the region of the sella turcica and, like stress-related conditions, can also disrupt the hypothalamic-pituitary-ovarian axis. Headache, vomiting, and vision loss or changes are some clinical features to look for; they are not apparent in this patient. Illicit drug use can also cause secondary amenorrhea and should be ruled out. Phenothiazines and opiates have been shown to suppress the activity of hypothalamic prolactin-inhibiting factor. Although this patient denies illicit drug use, this should also be further explored.

A 16-year-old girl presents after missing her period for the past 3 months. Onset of menses was at age 13, with initial irregularity for 6-8 months, and then consistency of occurring every 29 days. She has a serious boyfriend but repeatedly denies any sexual activity. She also denies any drug use. She has been healthy, without any stomach pains, vomiting, diarrhea, vaginal symptoms, or dysuria. She claims to eat quite well. Her weight is at the 75th percentile and height at the 25th percentile. On further questioning, she admits that she has actually been trying to lose weight and is exercising more to make the cheerleading team; she denies any binge eating, purging, or laxative use. There is a positive family history of diabetes. Question What is the most likely cause of secondary amenorrhea in the patient? 1 Pregnancy 2 Stress-related conditions 3 Diabetes mellitus 4 Craniopharyngioma 5 Illicit drug use

5 Patent ductus arteriosus The correct answer is patent ductus arteriosus. The ductus arteriosus, a connection between the pulmonary artery and aorta, is present during fetal life; it typically closes in the first few days of life. When the ductus arteriosus remains patent, it may lead to symptoms, depending on the size of the defect. Patients will have a continuous machine-like murmur; it is heard best at the left first and second intercostal spaces. The murmur of atrial septal defect would be heard as a loud systolic ejection murmur at the second and third intercostal spaces. The murmur of ventricular septal defect would be heard as a loud harsh holosystolic murmur at the left third and fourth intercostal spaces. Coarctation of the aorta may be associated with a continuous murmur heard in the back or over the anterior chest. However, the patient would also have hypertension present in the upper extremities and normal or low blood pressure in the lower extremities. The murmur associated with pulmonary stenosis would be heard as a loud harsh systolic murmur at the left second and third intercostal spaces.

A 2-week-old female infant presents for her scheduled newborn visit. The mother notes that the infant has been feeding poorly and seems to have difficulty catching her breath when crying. On examination, a continuous machine-like murmur is heard at the left first intercostal space. Question What is the most likely diagnosis? 1 Atrial septal defect 2 Ventricular septal defect 3 Coarctation of the aorta 4 Pulmonary stenosis 5 Patent ductus arteriosus

4 Repeat an abdominal X-ray in an hour. Explanation The critical issue in treating and managing swallowed foreign bodies is their size and their ability to pass through the pylorus. There are 5 areas of natural constriction in the pediatric age group where foreign bodies can get trapped: cricopharyngeal narrowing at the level of C6 the thoracic inlet aortic arch tracheal bifurcation hiatal narrowing When the foreign body has passed the pylorus, it will almost always work its way down and be passed in the stool. Of course, this is contingent on the object's presence prior to GI surgery or anomalies. Hence, the second X-ray of the abdomen to confirm its passage beyond the pylorus is warranted. Some may argue that the second X-ray is not necessary and the mother can take the child home and check the stool for the penny, only returning if it is not passed in 24 hours; the safest thing to do, however, is to confirm passage beyond the pylorus.

A 23-month-old boy is brought in by his distraught mother. Both are crying. It seems that while crawling on the floor, the child found a penny and put it in his mouth. Before the mother could get to him, he swallowed it. She put her finger in the baby's mouth and could not find anything. She immediately drove to the emergency department. It has been about an hour since the ingestion. When you see the child, he is just finishing a 4-ounce bottle. When the mother takes it away, he cries. His color is pink. Pulse ox is 98%. Chest sounds are clear and present bilaterally. Heart sounds are clear, and he has a sinus tachycardia. Abdomen is soft and flat. There is no tenderness. On rectal examination, brown stool is found, and there is no blood. A plain X-ray shows an object about the size of a penny present in a slightly distended stomach. The lungs are clear, and the heart shadow is in the midline and appropriate size and shape. Question What should occur next regarding evaluation and treatment of this patient? 1 Get an immediate surgical consult. 2 Place a nasogastric tube. 3 Intubate the child to prevent aspiration. 4 Repeat an abdominal X-ray in an hour. 5 Discharge the child and tell the mother to check his stool.

1 Mitral valve prolapse Explanation Mitral valve prolapse (MVP) is often asymptomatic. It can present with chest pain, palpitations, dyspnea, exercise intolerance, dizziness, and anxiety disorders. A physical exam often reveals a mid-to-late systolic click. If mitral regurgitation is also present, it can have a late systolic murmur as well. Murmurs are often best heard over the apex. The click is mobile, meaning that its timing varies with maneuvers that change the left ventricular volume. MVP is more common in thin girls/women, many of whom also have skeletal deformities, such as pectus excavatum or scoliosis. Aortic stenosis also has a systolic click, but it is typically heard earlier in systole than an MVP click. It is also best heard in the right upper sternum instead of the apex. Aortic stenosis typically presents with chest pain, syncope, and shortness of breath. Severe dehydration can cause dizziness, rapid heartbeat, and low blood pressure. It does not cause chest pain. Dehydration is not a long-standing condition. Prior to these symptoms occurring, the patient would have had a dry mouth, headache, tiredness, extreme thirst, decreased urine output, and many other issues. Atrial myxoma may present with symptoms of weight loss, fever, and malaise. An exam may reveal a diastolic tumor plop or diastolic murmur similar to mitral stenosis. Patients with atrial myxoma do not present with a systolic murmur or click. Atrial fibrillation can present with palpitations, chest pain, shortness of breath, lightheadedness, and fatigue, among others. The physical exam is where AF is distinguished from MVP. Patients with AF have an irregular heart rhythm and do not typically present with a murmur or clicks. An EKG and an echocardiogram are needed for diagnosis.

A 24-year-old woman with a past medical history of mild scoliosis presents with palpitations, occasional chest pain, and dizziness upon standing from a supine position. Her symptoms have been ongoing for a while, but the patient's parents finally convinced her to come be evaluated. On cardiac auscultation, a mobile mid-to-late systolic click and a late systolic murmur heard best at the apex is noted; no other abnormalities are found. Blood pressure was 112/68 mm Hg. Question What is the most likely diagnosis? 1 Mitral valve prolapse 2 Aortic stenosis 3 Dehydration 4 Atrial myxoma 5 Atrial fibrillation

2 Fluorescein staining Explanation The clinical picture is suggestive of ultraviolet keratitis (snow blindness). Slit lamp examination after instillation of fluorescein will show damage to surface cells on the cornea. Visual acuity should be done prior to any testing, but it does nothing to confirm the diagnosis. Testing extraocular movements (EOM) will not confirm the diagnosis. Everting the eyelids is generally done to look at the tarsal plates. Acute angle-closure glaucoma, tested with the Schiotz tonometer, would cause unilateral globe pain and a firm globe.

A 25-year-old man is on a skiing vacation. After a few hours of sleep the third night, he awakens with severe bilateral eye pain associated with lacrimation and photophobia. Question What procedure or ocular maneuver would give you the most information to assist in your diagnosis? 1 Visual acuity 2 Fluorescein staining 3 Test extraocular movements (EOMs) 4 Evert eyelids bilaterally 5 Schiotz tonometer evaluation

1 Discontinue ibuprofen. Explanation This patient has a history and findings consistent with acute (tubulo) interstitial nephritis (AIN) and should discontinue ibuprofen. AIN is an immune-mediated form of acute kidney injury (acute renal failure). Patients develop varying degrees of renal failure, which are characterized by changes in urine output, electrolyte imbalances, acidemia, and azotemia (elevations in serum creatinine with or without nausea, sleep disturbances, shakiness, etc.) in response to viral, bacterial, immunological, or pharmaceutical insults. A variety of antibiotics, ACE inhibitors, proton pump inhibitors, seizure medications, etc. are known to cause AIN. The development of AIN is not dependent on the dose of the medication given. Many infections, including HIV, EBV, and mumps, along with immunological diseases such as lupus and Wegener's granulomatosis can also cause AIN. In the case of this patient, no other triggers for AIN were noted, and ibuprofen is the likely cause. AIN is treated by removing the offending medication and avoiding its future use. If AIN occurs in the setting of an infection or immunologic or neoplastic process, that disease should be treated. Discontinuing the offending medication (if still in use) may cause resolution of illness in a couple of weeks. Patients with incomplete resolution may improve slowly over months. Symptoms, urine output, volume status, serum creatinine, and electrolytes should be monitored to evaluate the need for dialysis. Although he has an elevated serum creatinine, this patient lacks the traditional indications for dialysis, such as uncontrollable hyperkalemia, acidemia, volume overload, and uremia (azotemia + pericardial rub, encephalopathy, and/or asterixis). If renal failure does not improve within a few days of discontinuing the offending medication, patients with AIN can be started on prednisone therapy for a couple of weeks, to be tapered thereafter. For patients with AIN unresponsive to steroid therapy, immunomodulatory therapy such as cyclophosphamide (Cytoxan) can be considered to target the tubular and interstitial irritation that is occurring. Immunomodulatory therapy is premature in this case because other therapeutic maneuvers have not been attempted. Routine steroid therapy is not advised in the treatment of AIN. Cyclophosphamide therapy has many side effects, including neutropenia, and patients on this therapy require close monitoring for infections and decreasing blood counts.

A 30-year-old man presents to the ER with fever, malaise, and decreased urine output while taking ibuprofen for back pain. His temperature is 101°F, his blood pressure is 135/85 mm Hg without orthostatic change, and the remainder of his examination is normal. A post-void residual is normal, as is his renal ultrasound. His urine output is estimated at 1.5 L/day. His laboratory work shows: Serum creatinine: 2.0 mg/dL Serum potassium: 4.2 meq/L Serum bicarbonate: 23 meq/L Urinalysis: pH 6, trace protein, no blood, no casts, many white cells, no leukocyte esterase or bacteria Complete blood count: 5.0x10^3/mL with elevated eosinophils Fractional excretion urine sodium: 1.5% These values are unchanged after administration of 1.5 liters of normal saline. Question What is the initial treatment for your suspected diagnosis? 1 Discontinue ibuprofen. 2 Dialyze. 3 Prescribe steroids. 4 Prescribe cyclophosphamide (Cytoxan). 5 Prescribe antibiotics.

3 Fibrocystic condition Explanation The clinical picture is suggestive of a fibrocystic condition; this occurs commonly in women 30-50. The patient may have an asymptomatic mass or may discover a mass while the pain is occurring. The pain usually worsens during the premenstrual cycle. Fluctuations in size and the disappearance of masses are common with this condition. Bowen's disease is a form of squamous cell carcinoma. It typically presents as well-demarcated erythematous plaques with scaling on the skin surface; they are typically seen in sun-exposed areas. Phyllodes tumor is a fibroadenoma-like tumor. These tumors can become quite large due to their rapid growth; they may recur after excision. They commonly occur in women 40-50. Fibroadenoma is a common benign neoplasm seen in young women. The fibroadenoma is described as round, rubbery, discrete, relatively mobile, and non-tender. Fibrocarcinoma is a malignant tumor of the breast. The tumor usually consists of a non-tender firm or hard mass with poorly defined margins.

A 30-year-old woman presents with recurrent bilateral breast lumps and pain. She states that it seems to be worse during the last few days of her menstrual cycle; the lumps appear to get smaller after her cycle. She has felt different sized lumps in her breast that occur at the same time as the pain. On examination, several small, nodular lesions are noted in both breasts; they are freely movable. The axillary lymph nodes are unremarkable bilaterally. Question What is the most likely diagnosis? 1 Bowen's disease 2 Phyllodes tumor 3 Fibrocystic condition 4 Fibroadenoma 5 Fibrocarcinoma

5 Paroxetine Explanation Selective serotonin reuptake inhibitors (SSRIs) are the first-line treatment options for chronic, sustained treatment of panic disorder. Some SSRIs like paroxetine (Paxil) can be used in conjunction with a benzodiazepine, such as lorazepam or clonazepam, for short periods of time for acute panic attacks. Hydroxyzine is used primarily as an antihistamine and can be used for sleep. Buspirone is an anxiolytic used for anxiety. Phenobarbital is a barbiturate used to control seizures as well as anxiety.

A 31-year-old non-breastfeeding woman presents with recurrent episodes of severe palpitations, tachycardia, dyspnea, and impending dread that began shortly after the birth of her first child, which occurred about 4 weeks ago. Although the patient admits to very sporadic episodes of similiar symptoms prior, she was able to utilize meditation methods to resolve them. She has been attempting to use her techniques and unfortunately this has given her no relief from her symptoms. Question Ico-delete Highlights Based on her most likely diagnosis, what is best for the sustained treatment of her condition? 1 Lorazepam 2 Hydroxyzine 3 Buspirone 4 Phenobarbital 5 Paroxetine

1 Autoimmune Explanation The probable diagnosis in this patient is Graves' disease, which is autoimmune in etiology. Thyrotoxicosis occurs due to thyroid hormone excess, the etiology of which varies from Graves' disease, toxic multinodular goiter, toxic adenoma, thyroiditis, and even functioning metastasis. Symptoms of thyrotoxicosis include hyperactivity, irritability, dysphoria, heat intolerance and sweating, palpitations, fatigue and weakness, weight loss with increased appetite, diarrhea, polyuria, and menstrual irregularities. Graves' disease is the most common cause of thyrotoxicosis, an autoimmune disease in which thyroid-stimulating autoantibodies stimulate thyroid receptors to secrete thyroxine. It is more common in women and can be identified by radionucleotide scan, in which the whole thyroid shows increased uptake. Here, the patient has all Graves' disease characters: she is middle-aged with thyrotoxicosis characteristics, and her Tc-99 scan shows the characteristic Graves' pattern. Toxic multinodular goiter is another cause of thyrotoxicosis in which there are multiple nodules that can be felt with palpation, and it is shown to be hot or active with the radionucleotide scan. Hashimoto thyroiditis is an inflammatory condition that follows a viral infection. It is associated with a brief period of hyperthyroidism that is followed by hypothyroidism, which may persist throughout life. Factitious hyperthyroidism is due to the ingestion of thyroid hormones. Usually, this happens in nurses, physicians, or those who have access to medicine. It can be detected by measuring T3 and T4. It results from malingering. Toxic adenoma is a benign neoplastic nodule that actively secretes thyroxine.

A 35-year-old woman presents with a 2-month history of palpitation and nervousness. She mentions that she always feels hot, even if the weather is cold. Her menses have been irregular lately, and she has had no fevers recently. She was also told that her eyes are "weird looking." On examination, her blood pressure is 150/70 mm Hg, and her pulse is 89 beats per minute. Her eyes show exophthalmos, and she has lid lag when looking down. Thyrotoxicosis is suspected. What follows is the thyroid scan result: Refer to the image. Question What is the nature of this patient's illness? 1 Autoimmune 2 Hypersensitivity 3 Neoplastic 4 Inflammatory 5 Malingering

3 Hordeolum Explanation The correct answer is hordeolum (or stye), which is an infection that occurs at the lid margin. It is most often caused by bacteria and is treated with warm compresses and sometimes antibiotic ointment. An internal hordeolum is an infection of the meibomian gland that expands onto the lid conjunctiva. An external hordeolum is typically smaller than an internal one and found on the lid margin. Xanthelasma is a raised yellowish plaque in the skin of the eyelids. It is not painful and is classically associated with high cholesterol. A chalazion is a chronic inflammation of the meibomian gland inside the eyelid, not the lid margin, and is characteristically hard and nontender. Pinguecula is a nodule in the bulbar conjunctiva. Dacryocystitis is an inflammation of the lacrimal sac that leads to pain, swelling, and erythema around the tear sac.

A 35-year-old woman presents with a painful swelling of her left eyelid. On physical exam, there is tenderness to palpation and erythematous swelling present on the lid margin involving the eyelashes. Question What is the most likely diagnosis? 1 Xanthelasma 2 Chalazion 3 Hordeolum 4 Pinguecula 5 Dacryocystitis

1 Screening should be performed 8 - 10 years after the initial diagnosis. Explanation There is an increased risk of cancer in patients with ulcerative colitis. Surveillance colonoscopy with multiple biopsies is recommended every 1 to 2 years beginning 8 - 10 years after the diagnosis of pancolitis or 12 to 15 years after diagnosis of left-sided colitis according to guidelines of the American Gastroenterological Association.

A 38-year-old man was diagnosed with ulcerative colitis 7 years ago. He has been relatively stable on a course of daily sulfasalazine. The patient's initial diagnosis was made via colonoscopy, which he described as a traumatic experience. He is reluctant to undergo any other colonoscopic exams. Question Regarding screening colonoscopy for colon cancer in a patient with ulcerative colitis, which of the following statements is most accurate? 1 Screening should be performed 8 - 10 years after the initial diagnosis. 2 Screening should begin when the patient is 50 years old. 3 There is no role for screening colonoscopy. 4 Screening should be performed when the patient is 40 years old. 5 Screening should be performed 2-3 years after the initial diagnosis.

3 Rest and NSAIDs Explanation The clinical picture is suggestive of acute pericarditis. Most cases are due to viral infections with the treatment being rest and non-steroidal agents, e.g. aspirin or indomethacin. If this pericarditis progressed to tamponade, pericardiocentesis would be indicated. Symptoms of tamponade are not seen in this patient (dyspnea, elevated jugular venous pressure, hypotension, paradoxical pulse, and muffled heart sounds). Beta-blockers and calcium channel blockers are not indicated for treating pericarditis. Corticosteroids are usually given in cases unresponsive to rest and NSAIDs. Most cases of pericarditis are self-limiting and usually run their course from 1-3 weeks, but initial treatment consists of rest and NSAIDs.

A 39-year-old man presents with a 1-week history of severe chest pain. He states that the pain seems to worsen when he lies down. He describes the pain as radiating to the back and worsening when he takes a deep breath. His vital signs are as follows: blood pressure 124/84 mm Hg, respiratory rate 18/min, temperature 101°F, and pulse 74 beats per minute. On auscultation of the chest, you cannot distinguish an S1 or S2 but hear a scratching or grating sound. Question What is the first step in the treatment of this patient? 1 Pericardiocentesis 2 Beta-blockers 3 Rest and NSAIDs 4 Corticosteroids 5 Calcium channel blockers

3 Supracondylar humerus fracture Explanation Supracondylar fractures of the distal humerus are the most common elbow fractures in children age 2-12. The typical mechanism of this type of fracture is a fall on an outstretched arm, usually from a height. Proximal humerus and distal radius fractures will not show deformity at the elbow unless also associated with injury at the elbow. Fracture of the lateral epicondyle of the humerus accounts for 20% of elbow fractures in children. Fracture of the medial condyle of the humerus is also uncommon.

A 4-year-old girl presents after a fall from a 4-foot slide 30 minutes ago. She fell with her right dominant arm outstretched and her elbow fully extended. There was no loss of consciousness, but there was extreme pain. She immediately grabbed her right forearm and her right elbow. There is an obvious deformity at the elbow. Question What is the most likely diagnosis? 1 Proximal humerus fracture 2 Distal radius fracture 3 Supracondylar humerus fracture 4 Lateral epicondyle humerus fracture 5 Medial condyle humerus fracture

2 NSAIDs Explanation This patient is most likely in stage 1 of complex regional pain syndrome (CRPS). This disorder is rare; it causes instability of the vasomotor and autonomic components. CRPS is a disorder of a body region, most commonly the extremities, that is characterized by pain, swelling, limited range of motion, vasomotor instability, skin changes, and even patchy bone demineralization. Frequently, symptoms will begin after the patient has sustained an injury, had surgery, or experienced a vascular event, such as a stroke. 35% of patients with CRPS report not having a distinct precipitating event. Stage 1 of CRPS characteristically has findings such as pain in the limb (burning/throbbing), diffuse and uncomfortable aching, sensitivity to touch or cold, or even localized edema. The distribution of pain is not compatible with a single peripheral nerve, trunk, or root lesion. There will also be signs of variable vasomotor disturbances that result in altered color and temperature. A radiograph completed at this stage will most likely be normal, but it could show signs of patchy demineralization. Early treatment intervention provides the best prognosis for CRPS. The patient being in an early stage is also a promising component. First-line treatment for mild cases is nonsteroidal anti-inflammatory drugs, which are commonly referred to as NSAIDs. Naproxen is commonly considered in this scenario. Calcitonin may be considered in patients who still have mild or moderate symptoms despite the use of NSAIDs. A meta-analysis of the results of five clinical trials showed that it is effective in treating pain caused by CRPS, but calcitonin is not considered first-line treatment in patients. Sulfasalazine, TNF inhibitors, and etanercept are indicated in the treatment of inflammatory autoimmune conditions such as rheumatoid arthritis, psoriatic arthritis, ankylosing spondylitis, and ulcerative colitis.

A 40-year-old woman presents with a 7-day history of pain in her right arm. The patient denies any trauma or injury to this extremity just prior to the pain starting, but she does admit to having a Colles' fracture in this arm around 2 months ago. She denies any injury to her back, neck, or any other musculoskeletal system prior to the event of pain. She describes the pain as burning and throbbing; there is an extremely diffuse and uncomfortable aching accompanying it. She further states that this limb has become extremely sensitive to touch and cold, and it appears somewhat more swollen than her left arm. The patient is very upset because she does not know why her arm is so painful when she has not done anything to it. She is a non-smoker, does not drink, and exercises 3 times a week. Physical examination of the extremity reveals a slightly cyanotic, mottled right arm with generalized pain of the entire extremity. Pulses are faint (1+), and ROM is limited. Question Keeping in mind the most likely diagnosis, what is the most appropriate first-line therapy for the patient's signs and symptoms? 1 TNF inhibitors 2 NSAIDs 3 Sulfasalazine 4 Etanercept 5 Calcitonin

3 Excision of thrombosed external hemorrhoid Explanation The clinical case described most likely represents a patient with a thrombosed external hemorrhoid. External hemorrhoidal thrombosis is not infrequent and can be seen in patients with no prior history of hemorrhoidal disease. The cause is thought to be from elevated venous pressures related to excessive straining with constipation and physical exertion following a bout of diarrhea or change in diet. It is an acute painful event with the pain generally lasting 7-14 days and resolving with the resolution of the thrombosis. The clinical presentation is usually as described above, with acute swelling and pain located at or around the anal verge. When the overlying skin of the thrombosed hemorrhoid is eroded, bleeding occurs. The pain usually lasts for a few days and then spontaneously resolves. The swelling may take several weeks to resolve. Treatment depends on the time of presentation and the severity of the condition. Acute thrombosed external hemorrhoid presenting within 72 hours of the onset of symptoms should be excised. Excision of the thrombosed external hemorrhoid can be performed under local anesthesia and is associated with a low recurrence and complication rate. Patients presenting after 72 hours of onset of symptoms should be managed by conservative/symptomatic medical therapy. Antibiotics, immediate surgical drainage, colonoscopy, and sphincterotomy are not the treatment options for thrombosed external hemorrhoid.

A 45-year-old man presents with a 24-hour history of severe anal pain and swelling. The pain started after straining at defecation and has worsened over the course of the day. There is no history of fever. Examination of the anal area reveals a swollen ecchymotic mass in the perianal skin, very close to the anal verge. What is the treatment of choice for this condition? 1 Immediate surgical drainage in the operating room 2 Antibiotics 3 Excision of thrombosed external hemorrhoid 4 Immediate colonoscopy 5 Internal sphincterotomy

4 Cushing syndrome Explanation Cushing syndrome may develop as a result of small cell carcinomas secreting ectopic adrenocorticotropic hormone. Patients may complain of bruising easily. On examination, patients may have red-purplish striae, especially over the abdomen, in addition to the typical moon facies and truncal obesity. They may have muscle wasting, especially of the proximal limb girdle muscles. Laboratory investigations reveal hypokalemia and high plasma ACTH, as well as increased serum and urine cortisol. Lambert-Eaton syndrome is an immune-mediated disorder of neuromuscular transmission resulting in impaired release of acetylcholine from nerve terminals. Patients note weakness of the scapular and pelvic girdle muscles and the resultant difficulties in performing activities such as climbing stairs. Other symptoms (e.g., dry mouth, paresthesias, and sexual impotence due to autonomic dysfunction) may also be present. On examination, there is proximal muscle weakness. Deep tendon reflexes are decreased or absent. Symptoms of Pancoast syndrome include one or more clinical signs that are due to compression or involvement of the brachial plexus and the cervical sympathetic nerves. Clinical signs include weakness and atrophy of the muscles of the hand, ipsilateral ptosis, anhidrosis, and miosis (Horner's syndrome) that are due to compression or involvement of the brachial plexus and the cervical sympathetic nerves. Pancoast syndrome is more common in non-small-cell lung cancer. Horner's syndrome is due to a bronchogenic carcinoma tumor mass extending to the sympathetic chain. Horner's syndrome symptoms include miosis, ptosis, enophthalmos, and hemianhidrosis. Patients may also have atrophy of hand muscles. The syndrome of inappropriate antidiuretic hormone secretion is a paraneoplastic syndrome associated with bronchogenic carcinoma; there is excessive water reabsorption by the kidneys due to elevated antidiuretic hormone. Patients may complain of feeling weak and lethargic. Other symptoms include confusion and coma. Laboratory tests reveal hyponatremia, high urine sodium, low serum osmolality, and high urine osmolality in a euvolemic patient.

A 55-year-old man presents with a 2-day history of hemoptysis; he has a chronic cough that he attributes to his 40 pack-year history of cigarette smoking. He also reports that he has been bruising easily. On examination, he has moon facies and truncal obesity with wasting of his shoulder muscles. He has several red-purple striae over his anterior abdominal wall. The lungs are clear to auscultation. Laboratory results are as follows: hemoglobin 12 g/dL, white blood cell count 8 x 103/mm3, platelets 160 x 103/mm3, sodium 138 mmol/L, and potassium 3.0 mmol/L. Tuberculin skin test is negative. Sputum cytology is positive for carcinoma. Question What syndrome commonly associated with lung cancer is the most likely diagnosis? 1 Lambert-Eaton myasthenic syndrome 2 Pancoast syndrome 3 Horner's syndrome 4 Cushing syndrome 5 Syndrome of inappropriate antidiuretic hormone secretion

3 Mycoplasma pneumoniae Explanation Mycoplasma pneumoniae is a pathogen most frequently associated with atypical community-acquired pneumonia. Atypical pneumonia caused by M. pneumoniae is more common in school children and young adults. The bacterium usually causes asymptomatic infection or mild respiratory disease. Bronchopneumonia involving 1 or more lobes develops in 3-10% of infected persons. A non-specific serological test using human O group RBCs can be used for detecting cold agglutinin titers in pneumonia caused by Mycoplasma pneumoniae. High levels are seen in about 50-60% of untreated patients with M. pneumoniae pneumonia. In a patient strongly suspected of having M. pneumoniae infection, the presence of cold agglutinins with a significant titer is reasonably supportive evidence for a diagnosis. Even though non-specific and crude, a positive cold agglutination test is helpful in diagnosing M. pneumoniae pneumonia in adults. A significant level of cold agglutinins is not seen in infections by other microorganisms listed. Chlamydia pneumoniae is another important agent of community-acquired atypical pneumonia. This is an obligate intracellular organism capable of persistent latent infection. Most cases of pneumonia are mild and the clinical spectrum resembles M. pneumoniae infection. Diagnosis of C. pneumoniae pneumonia is by antigen detection using EIA, direct immunofluorescence, or molecular methods. A cold agglutination test is negative. Legionella pneumophila is an aerobic, fastidious, Gram-negative bacillus associated with community-acquired pneumonia. Several serotypes have been identified. Outbreaks of infections have been associated with condensers, cooling towers, respiratory therapy equipment, showers, whirlpools, and water faucets. Laboratory diagnosis is by demonstration of the organisms in clinical specimens such as sputum, bronchial lavage, lung biopsy by fluorescent antibody test, isolation of the pathogen by culture on specialized media, by urinary legionella antigen test, and by demonstration of specific antibodies in serum by ELISA or indirect fluorescent assay. Cold agglutinins are not a feature of this infection. Coxiella burnetii belongs to the Rickettsial group of organisms. It causes Q fever, a zoonotic disease transmitted by ticks among domestic livestock and other animals. Though ticks are important vectors and transmit the infection among animals, they do not seem to be important in the transmission of human infection. Chlamydia psittaci causes psittacosis, a zoonotic disease acquired from birds. Infection can produce a wide spectrum of diseases ranging from mild influenza-like illness to fatal pneumonia. Inhalation of infected dried bird excreta is the common mode of infection. Human infections are mostly occupational and seen in poultry workers, pet shop owners, and veterinarians.

A 45-year-old man presents with a 3-week history of fever, malaise, and cough. The illness began with a low-grade fever, headache, sore throat, and non-productive cough. He tried symptomatic treatment with over-the-counter medicines and did not take any antibiotics. His condition worsened and he developed expectoration of non-blood stained sputum, shortness of breath, and chest soreness. There was no history of exposure to birds or any new environment. Chest auscultation detected scattered rales and expiratory wheezes. Chest X-ray showed bilateral diffuse infiltrates and consolidation of right lower lobe. A Gram stain of sputum showed the presence of leukocytes and normal flora. Routine culture for bacterial pathogens grew only normal flora. The microbiology laboratory was not equipped for isolation of fastidious organisms. Considering the presumptive clinical diagnosis of atypical pneumonia, a supportive non-specific serological test was done to detect cross-reacting antibodies to human erythrocytes. The test results showed significant titer of the antibodies. Question What is likely to be the causative agent in the above patient? 1 Chlamydia pneumoniae 2 Legionella pneumoniae 3 Mycoplasma pneumoniae 4 Coxiella burnetii 5 Chlamydia psittaci

5 Polysomnography Explanation The patient's presentation is most consistent with obstructive sleep apnea, a diagnosis that is made definitively by polysomnography. Polysomnography involves direct sleep observation with the simultaneous measurement of multiple parameters, including electrical brain activity (as measured by electroencephalogram), eye movements, skeletal muscle contractions (as measured by electromyogram), heart rhythm (as measured by electrocardiogram), and respiratory effort. Obstructive sleep apnea is defined as partial or complete upper airway obstruction caused by anatomic collapse of pharyngeal structures. Patency of the upper airway is partially (hypopnea) or completely (apnea) lost during sleep. This results in progressively increasing inspiratory effort against an obstructed glottic opening, which causes negative intrathoracic pressures to build. At some point, brain stem reflexes result in an altered sleep state, which prompts reopening of the airway. This may awaken the patient or be observed as a "gasping" respiratory effort. The patient's symptoms are characteristic of the disorder; snoring is indicative of obstructed airflow, and daytime somnolence is a function of disrupted sleep. Some patients will report a headache, which may be related to changes in blood oxygen and carbon dioxide levels and/or fluctuations in blood pressure that accompany periodic apneic episodes during sleep. Long-term chronic hypoxemia and (perhaps) hypercarbia may result in pulmonary vasoconstriction and right heart strain. Polysomnography is useful in diagnosing other disorders of sleep as well. For example, restless leg syndrome or periodic limb movements may be documented via increased skeletal muscle activity during sleep. Alternations in depth of sleep may also be observed and may be related to caffeine, alcohol, or medication use. Neither Holter monitoring nor echocardiograms have a role in the diagnosis of a sleep disorder. Electroencephalograms, electromyograms, and electrocardiograms alone are unlikely to allow for the correct diagnosis of a sleep disorder. Furthermore, they are components of polysomnography, making it a more robust test of sleep dysfunction.

A 49-year-old man presents with chronic fatigue. He states that despite obtaining 7-9 hours of sleep nightly, he wakes up feeling unrested and sometimes with a headache. Occasionally, he falls asleep in the middle of the day. He is not sure if he snores at night, but he has been told that he snores when he is napping. He reports no other changes in general condition. His other medical problems are hypertension and hypercholesterolemia, for which he takes metoprolol and simvastatin, respectively. Physical exam reveals a moderately overweight man in no apparent distress. Blood pressure, heart rate, and resting oxygen saturation are within normal limits. Question Based on the patient's symptoms, what diagnostic study would be most useful? 1 24-hour Holter monitoring 2 Echocardiogram 3 Electrocardiogram 4 Electroencephalogram 5 Polysomnography

4 Chronic bronchitis Explanation The clinical picture is suggestive of chronic bronchitis. Chronic bronchitis is defined by a clinical history of productive cough for 3 months of the year for 2 consecutive years. Smoking is the leading cause. The principle pathologic feature is airway injury and narrowing, hypertrophy of the airway mucous glands, infiltrate of inflammatory cells, and loss of ciliated epithelium. The cough produces thick, often purulent sputum because of the ongoing local inflammation and the high likelihood of bacterial colonization and infection. The increased mucous production and defective mucociliary escalator function lead to inspiratory and expiratory crackles. On imaging, common findings are hyperinflation of lung volumes, relatively depressed diaphragm, and parallel linear densities of thickened bronchial walls known as tram track lines. Asthma produces wheezing lung sounds. They are not heard in this patient. A productive cough is a rare clinical manifestation of emphysema, and the CXR is more descriptive of bronchitis. Pulmonary edema may present with mild exertional dyspnea or a nonproductive cough, although a frothy or blood-tinged sputum may be seen. Pulmonary fibrosis is a restrictive lung disease with the clinical features of progressive dyspnea. It is typically accompanied by a dry, persistent hacking cough.

A 53-year-old woman presents with a 2-year history of chronic cough. The cough produces large volumes of grossly purulent sputum. She has a history of recurrent respiratory infections; they resulted in 5 hospitalizations in the past year. She also had similar complaints during the previous year. Shortness of breath limits her daily activity considerably. Upon pulmonary examination, bilateral breath sounds are audible, with inspiratory and expiratory crackles at the lung bases. Chest X-rays reveal increased lung volumes, non-flattened diaphragm, and thickened bronchial walls. Question What is the initial diagnosis? 1 Asthma 2 Emphysema 3 Pulmonary edema 4 Chronic bronchitis 5 Pulmonary fibrosis

5 Acute myocardial infarction Explanation The patient has several risk factors for coronary heart disease including diabetes, hypertension, and cigarette smoking. A patient presenting with signs and symptoms of ischemia and new ECG changes meets the criteria for acute myocardial infarction (AMI). Stable angina, unstable angina, and AMI constitute a spectrum of diseases related to underlying atherosclerosis. Stable angina causes chest pain that is usually predictable, is due to physical exertion, and is relieved by rest. Chest pain that becomes more frequent, occurs more often, or occurs at rest is unstable angina. Unstable angina is differentiated from AMI by ECG and biochemical marker changes (Troponin, CK-MB). Diabetes mellitus (DM) is an independent risk factor for atherosclerosis. The risk of myocardial infarction (MI) in a patient with diabetes is the same risk as someone without diabetes who has had a previous MI. The risk of death from cardiac events is also the same between the 2 groups. Other important contributors to atherosclerosis and ischemic heart disease include dyslipidemia, cigarette smoking, obesity, and hypertension, which contribute to the risk of MI. However, when evaluated independently, DM is a greater risk factor. Patients with diabetes mellitus should be advised to stop smoking and aggressively control other risk factors, such as glucose, hypertension, and dyslipidemia, in order to reduce the risk of ischemic heart disease. Patients with pericarditis report that pain is related to position and worse with inspiration. The ECG shows diffuse ST elevation. Patients with pulmonary embolism may present with acute onset chest pain and dyspnea, but will not have the characteristic ECG changes. Chest pain associated with costochondritis is reproducible with palpation of the affected area and is not associated with dyspnea or ECG changes.

A 54-year-old man presents with chest pain. He has a past medical history of hypertension and diabetes mellitus. The pain is located in the middle of his chest and radiates to his jaw. The pain began about 20 minutes ago, and he rates the pain as a 10 on a 0-10 point scale, with 10 being the worst pain he has ever felt. He has had 3 similar episodes, but they have always resolved after 5 minutes or so of rest. He has smoked 1 pack of cigarettes a day for the past 36 years. He drinks 2 or 3 beers on Friday nights. Review of systems (ROS) is positive for diaphoresis, acute dyspnea, and sense of impending doom. ROS is negative for fever, chills, and malaise. Physical exam shows an obese, middle-aged man in moderate distress. BP is 148/80, pulse is 100, and respirations are 26. Except for tachycardia and tachypnea, heart and lung exams are normal. He has no pedal edema. Electrocardiogram (ECG) shows ST elevation in leads II, III, and AVF; this is a new finding when compared to an ECG from 3 months ago. Question What is the diagnosis? 1 Pericarditis 2 Stable angina 3 Pulmonary embolism 4 Costochondritis 5 Acute myocardial infarction

1 A progestin during times of amenorrhea if the patient was not already on contraceptives Explanation The correct response is administration of a progestin during times of amenorrhea if the patient was not already on contraceptives. This patient's long period of amenorrhea followed by postmenopausal bleeding is suggestive of progression from endometrial hyperplasia to endometrial cancer. Various types of hyperplasia (e.g., simple, complex, atypical simple, and atypical complex) can be treated with progestin, which inhibits and reverses endometrial hyperplasia. Depot medroxyprogesterone acetate is considered a type of progestin that will treat endometrial hyperplasia and prevent progression to cancer. Total avoidance of DMPA throughout the patient's life would not have protected her uterus. Oral contraceptives contain a progestin component that helps counter the endometrial overgrowth that leads to hyperplasia and cancer development. In fact, oral contraceptive use is considered a protective factor for the development of endometrial cancer. The avoidance of oral contraceptives in this patient may have hastened the development of her cancer. The patient's diabetes is a risk factor for the development of endometrial cancer, especially if combined with obesity, but the type of treatment for diabetes (oral medication or insulin injections) has not been clearly associated with better or worse outcomes in endometrial cancer. An improved degree of glucose control indicates a decreased risk of endometrial cancer compared to poor glucose control in a diabetic. Insulin tends to provide better glucose control if the patient is compliant with the treatment. In this patient who already admits poor compliance with oral diabetes medication (metformin), it is anticipated that her compliance would be similar or worse with insulin injections. No firm conclusion can be drawn that the type of diabetes treatment would have prevented endometrial cancer. Yearly Pap tests may or may not have detected abnormalities in pathology; the Pap is not considered a reliable screening test for detecting endometrial cancer. If atypical glandular cells of undetermined significance are detected on a Pap, then definitive evaluation (e.g., a dilatation and curettage or endometrial biopsy) should be pursued. Having regular Pap tests would not have prevented the development of endometrial cancer, but they potentially could have detected an abnormality at an earlier time.

A 54-year-old woman with diabetes mellitus presents with a 3-week history of vaginal spotting. An endometrial biopsy is performed and the pathology indicates endometrial cancer. The patient has a history of 12 years of amenorrhea and considers herself postmenopausal. She denies pelvic pain and cramping. She cannot identify any health changes that relate to the new vaginal bleeding. She has not seen a medical provider for preventive services for over 5 years due to a lack of health insurance. She is currently on metformin 500 mg TID; the patient admits poor compliance with the second and third doses each day. She has no known allergies, and her past medical history is significant for a cholecystectomy and tonsillectomy. The patient does not smoke or drink alcoholic beverages and denies illicit drug use. Menarche was at age 15; her menses were generally regular in her teens and 20s. She was amenorrheic while using depot medroxyprogesterone acetate, then menses returned but were irregular in frequency in her 30s. She believes her LMP to have been around age 42. She denies significant dysmenorrhea, menorrhagia, or premenstrual syndrome throughout her menstrual history. She is a G8P6Ab2, with her first child delivered at age 19; she had 6 spontaneous vaginal births with two first-trimester spontaneous abortions. She has had 9 sexual partners and has no history of sexually transmitted diseases. Previous methods of contraception included oral contraceptives (briefly) in her 20s and depot medroxyprogesterone acetate (DMPA) for 5 years in her early 30s. She discontinued the use of contraceptives in her mid-30s and has rarely been sexually active since. Question What intervention would have had the best effect in preventing this patient's development of endometrial cancer? 1 A progestin during times of amenorrhea if the patient was not already on contraceptives 2 Avoidance of depot medroxyprogesterone acetate (DMPA) injections for contraception 3 Avoidance of the oral contraceptives 4 Treatment of her diabetes with insulin instead of oral medication 5 Yearly Pap tests over the last 5 years

4 Anemia workup and treatment now, when hemoglobin is 11.5 g/dL Explanation The correct response is anemia workup and treatment now, when hemoglobin is 11.5 g/dL. Patients with declining renal function may have anemia for a variety of reasons, including decreased erythropoietin production, iron deficiency, and parathyroid dysfunction-related erythropoietin resistance. Anemia should be evaluated if hemoglobin (Hgb) <12 g/dL in women and prepubertal patients, and if Hgb <13 g/dL in adult men (according to the 2012 KDIGO guidelines). Not treating anemia can cause a variety of symptoms, including fatigue and weakness. Markers of iron deficiency include an iron saturation <20% and a ferritin <100 ng/mL. Treatment of iron deficiency should start with oral iron; intravenous iron can be given if patients do not respond to oral therapy. Intact parathyroid hormone levels should be <110 pg/mL in patients with chronic kidney disease stages <5 (not requiring dialysis). Treatment involves giving oral vitamin D3 to suppress parathyroid levels to a target level of 70-110 pg/mL. Using the Modification of Diet in Renal Disease (MDRD) equation to estimate glomerular filtration rate (GFR), this 55-year-old Caucasian woman with a creatinine of 1.5 mg/dL has an estimated GFR of 38 mL/min/1.73 m2. This moderately reduced GFR is a marker of kidney disease. Patients who have such markers for >3 months are considered to have kidney disease and should be referred to a nephrologist for assistance in managing other abnormalities (e.g., anemia, calcium, and phosphorus disorders) that can occur at this stage of the disease. Patients whose GFR is in the 30-59 mL/min/1.73m3 range are considered to have Stage 3 chronic kidney disease (patients who have a GFR below 60 are recommended to be referred, whereas below 30 it is compulsory to be referred).

A 55-year-old Caucasian post-menopausal woman comes to your office for follow-up of her hypertension, which she has had for 10 years. Blood pressure today is 130/80 mm Hg. You review her lab values and note that her serum creatinine is 1.5 mg/dL and her hemoglobin (Hgb) is 11.5 g/dL. Question What is the next best step in the management of this patient? 1 Anemia workup and treatment when her hemoglobin <11 g/dL 2 Anemia workup and treatment when her hemoglobin is 9 g/dL 3 Anemia workup and treatment when her hemoglobin is 8 g/dL 4 Anemia workup and treatment now, when hemoglobin is 11.5 g/dL 5 Anemia workup and treatment when her hemoglobin is 12 g/dL

1 Actinic keratosis Explanation The clinical picture is suggestive of actinic keratosis, which consists of single or multiple, discrete, dry, rough, adherent, scaly lesions that occur on the habitually sun-exposed skin of adults. They have a rough sandpaper-like texture to them, and they are usually described as "better felt than seen." Squamous cell carcinoma appears as a sharply demarcated, scaling, or hyperkeratotic macule, papule, or plaque. Solitary or multiple lesions are pink or red in color; they have slightly scaling surfaces, small erosions, and can be crusted. Solar lentigo lesions are strictly macular and 1-3 cm in diameter, but they can be as large as 5 cm. They appear as light yellow, light brown, or dark brown; they are uniformly mixed in color. Seborrheic keratosis consists of benign plaques that are 3-20 mm in diameter; their color can range from beige to brown or even black. They feel greasy and have a "stuck on" appearance. Onset is rare before age 30. Dermatofibromas are benign skin growths; they are typically found on the legs and range in size from about 0.5-2 cm. They can be described as hard papules that may appear in a variety of colors, but they are usually brownish to tan.

A 55-year-old man presents with lesions on the top of his head. He noticed them about 6 months ago and did not think much about them until more of them appeared. Physical examination of the scalp shows that he is slightly balding and has scattered multiple lesions that appear to be <1 cm in size. The lesions are yellow-brown, dry, and scaly. Upon palpation, the lesions have a rough, coarse texture and are tender. Question What is your initial diagnosis? 1 Actinic keratosis 2 Squamous cell carcinoma 3 Solar lentigo 4 Dermatofibroma 5 Seborrheic keratosis

5 Syndrome of inappropriate antidiuretic hormone secretion (SIADH) Explanation The syndrome of inappropriate antidiuretic hormone secretion (SIADH) is a paraneoplastic syndrome associated with bronchogenic carcinoma. There is excessive water reabsorption by the kidneys due to the elevated antidiuretic hormone (ADH). Patients may report feeling weak and lethargic. Other symptoms include confusion and coma. Laboratory tests reveal hyponatremia, high urine sodium, low serum osmolality, and high urine osmolality in a euvolemic patient. In hypertrophic pulmonary osteoarthropathy (HPO), there is formation of new subperiosteal cancellous bone at the distal ends of long bones. Patients present with periarticular pain, polyarthralgia, and painful swellings at the wrists, knees, elbows, and ankles. On examination, there is clubbing of the fingers and toes. There may also be localized articular erythema, tenderness, swelling, and even effusions. X-rays may reveal subperiosteal bone formation with periosteal elevation seen as thickening and detachment of the periosteum. Lambert-Eaton myasthenic syndrome is an immune-mediated disorder of neuromuscular transmission resulting in impaired release of acetylcholine from nerve terminals. Patients complain of weakness of the scapular and pelvic girdles muscles and the resultant difficulties in performing activities such as climbing stairs. Other symptoms such as dry mouth, paresthesia, and sexual impotence due to autonomic dysfunction may also be present. On examination, there is proximal muscle weakness. Deep tendon reflexes are decreased or absent. Horner's syndrome is due to a bronchogenic carcinoma tumor mass extending to the sympathetic chain. Horner's syndrome symptoms include miosis, ptosis, enophthalmos, and hemi anhidrosis. Patients may also have atrophy of hand muscles. Cushing syndrome may develop as a result of the small cell carcinomas secreting ectopic adrenocorticotropic hormone (ACTH). Patients may report bruising easily. On examination, they may have red-purplish stria (especially over their abdomen) in addition to the typical moon facies and truncal obesity. They may also have muscle wasting, especially of the proximal limb girdle muscles. Laboratory investigations reveal hypokalemia, high plasma ACTH, increased serum, and urine cortisol.

A 57-year-old woman presents to the hospital with a 2-week history of feeling weak. She is an artist who has been chain smoking for 30 years. On examination, she is kempt and wasted but not dehydrated. Her BP is 116/70 mm Hg. Laboratory investigations reveal plasma sodium of 122 mEq/L with a high urine osmolality. A chest radiograph reveals a rounded opacity in the right lung field. Question What is the most likely diagnosis? 1 Lambert-Eaton myasthenic syndrome 2 Hypertrophic pulmonary osteoarthropathy 3 Horner's syndrome 4 Ectopic adrenocorticotropic hormone (ACTH) secretion 5 Syndrome of inappropriate antidiuretic hormone secretion (SIADH)

4 Anti-DNase B serology Explanation The correct answer is anti-DNase B serology to identify post-streptococcal glomerulonephritis. Group A β-hemolytic streptococci pharyngitis may result in the delayed complication of post-streptococcal glomerulonephritis 10-14 days after the infection. Patient presentations may range from subclinical symptoms to acute nephritic syndrome as streptococci may produce streptolysin, DNase, and hyaluronidase that lead to tissue destruction and disseminate infection. Serology testing to identify antibodies to these exoenzymes can aid in the diagnosis by demonstrating indirect evidence of infection. Confirmation may require serial antibody draws that reveal a rise in titer levels above the baseline. Urine culture and sensitivity would be appropriate if the clinical picture only entailed fever and flank tenderness in the presence of pyuria and hematuria, suggesting a urinary tract infection. That diagnosis does not explain the presence of proteinuria with renal tubular epithelial cells and casts. These indicate intrinsic kidney damage, which is not seen in urinary tract infections. Further serology testing is indicated in the post pharyngitis period. Urine cytology is ordered in the presence of gross or microscopic hematuria, which is often painless, to identify malignant cells in the urinary tract. This patient does not fit the epidemiologic profile or clinical presentation of malignancy to warrant cytology testing. Erythrocyte sedimentation rate can aid in detecting an inflammatory response, such as nephritis, but it lacks specificity to identify the infectious process. Urine protein electrophoresis is indicated to identify abnormal levels of free monoclonal light chains (Bence Jones protein) from immunoglobulins in cases of myeloma.

A 6-year-old boy presents with a fever, malaise, and dark urine. His mother states that he missed school earlier this month with a sore throat. On examination, his blood pressure is 120/88 mm Hg, pulse is 82/min, temperature is 100.6°F, and respirations are 16/minute. On physical assessment, the patient appears ill with only mild costovertebral angle tenderness noted. Urinalysis reveals the following: Urinalysis Result Specific gravity 1.00 pH 5.2 protein + 1 blood + 2 glucose negative ketones negative bilirubin negative urobilinogen negative nitrates negative leukocyte esterase + 1 Microscopic examination reveals RBCs, renal tubular epithelial cells, RBC casts, and granular casts. Question Ico-delete Highlights Considering the most likely diagnosis, what laboratory test will help determine the most likely etiology? 1 Urine for culture and sensitivity 2 Urine cytology 3 Erythrocyte sedimentation rate 4 Anti-DNase B serology 5 Urine protein electrophoresis

1 Lambert-Eaton myasthenic syndrome Explanation Lambert-Eaton myasthenic syndrome is an immune-mediated disorder of neuromuscular transmission; it causes impaired release of acetylcholine from nerve terminals. Patients present with weakness of the scapular and pelvic girdle muscles and the resultant difficulties in performing activities (e.g., climbing stairs). Other symptoms (e.g., dry mouth, paresthesia, and sexual impotence) due to autonomic dysfunction may be present. On examination, there is proximal muscle weakness. Deep tendon reflexes are decreased or absent. In hypertrophic pulmonary osteoarthropathy (HPO), there is a formation of new subperiosteal cancellous bone at the distal ends of long bones. Patients present with periarticular pain, polyarthralgia, and painful swellings at the wrists, knees, elbows, and ankles. On examination, there is clubbing of the fingers and toes. There may also be localized articular erythema, tenderness, swelling, and even effusions. X-rays may reveal subperiosteal bone formation, along with periosteal elevation seen as thickening and detachment of the periosteum. Horner syndrome is due to a bronchogenic carcinoma tumor mass extending to the sympathetic chain. Horner syndrome symptoms include miosis, ptosis, enophthalmos, and hemi anhidrosis. In addition, the patients may also have atrophy of hand muscles. Cushing's syndrome may develop as a result of small cell carcinomas secreting ectopic adrenocorticotropic hormone (ACTH). Patients may complain of bruising easily. On examination, they may have red-purplish striae (especially over their abdomen) in addition to the typical moon facies and truncal obesity. They may also have muscle wasting, especially of the proximal limb girdle muscles. Laboratory investigations reveal hypokalemia, high plasma ACTH, increased serum, and urine cortisol. The syndrome of inappropriate antidiuretic hormone secretion (SIADH) is a paraneoplastic syndrome associated with bronchogenic carcinoma. There is excessive water reabsorption by the kidneys due to the elevated antidiuretic hormone (ADH). Patients may complain of feeling weak and lethargic; other symptoms include confusion and coma. Laboratory tests reveal hyponatremia, high urine sodium, low serum osmolality, and high urine osmolality in a euvolemic patient.

A 60-year-old man presents with a 1-month history of weakness, an inability to climb stairs, and erectile dysfunction. He is not on any medications and has no prior admissions. He has smoked 2 packs of cigarettes a day for the past 45 years. Noted abnormalities on examination include strength in his shoulder as well as in his pelvic girdle muscles at only a 2/5. All deep tendon reflexes are significantly decreased. Question What is the most likely diagnosis? 1 Lambert-Eaton myasthenic syndrome 2 Hypertrophic pulmonary osteoarthropathy 3 Horner syndrome 4 Ectopic adrenocorticotropic hormone (ACTH) secretion 5 Syndrome of inappropriate antidiuretic hormone secretion (SIADH)

2 Defibrillation Explanation The above patient has EKG findings of ventricular fibrillation. Defibrillation acts to briefly stop all electrical activity of the heart and can potentially terminate ventricular fibrillation. Chances of survival from ventricular fibrillation can drop by up to 10% for every minute the patient goes without defibrillation. Though CPR is an important intervention in the management of ventricular fibrillation, CPR alone will not treat ventricular fibrillation. However, the use of CPR between shocks increases the chance of survival compared to patients who receive defibrillation without the use of CPR between shocks. Epinephrine may be used in the treatment of ventricular fibrillation for its vasoconstrictive properties. It is not as effective as defibrillation. Vasopressin may be used in place of epinephrine in the treatment of ventricular fibrillation. It is not as effective as defibrillation. Amiodarone may be used in the treatment of ventricular fibrillation for its antiarrhythmic properties. There is currently no evidence that it increases patient survival.

A 62-year-old man collapses in the emergency department. The patient is connected to a cardiac monitor that shows the presence of an irregular deflection of varying amplitude with rapid rate and no identifiable P waves, QRS complexes, or T waves. Question What intervention has the greatest impact on this patient's survival? 1 CPR 2 Defibrillation 3 Epinephrine 4 Vasopressin 5 Amiodarone

2 Benign positional vertigo Explanation Benign positional vertigo (BPV) is the most likely cause of this patient's vertigo. BPV is not typically associated with hearing loss. Patients have brief episodes of vertigo with positional changes, typically when turning over in bed. This patient had a positive response to the Dix-Hallpike maneuver, which confirms the diagnosis. BPV is due to deposition of calcium debris in the semicircular canals. Medications such as diazepam or meclizine as well as Canalith Repositioning Procedure (Epley maneuver) are used to treat the condition. The latter is a series of head rotational positions intended to relocate free-floating particles in the semicircular canals. An acoustic neuroma (sometimes termed a neurilemmoma or schwannoma) is a benign (non-cancerous) tissue growth that arises on the eighth cranial nerve leading from the brain to the inner ear. Acoustic neuromas usually grow slowly over a period of years. They expand in size at their site of origin and, when large, can displace normal brain tissue. The brain is not invaded by the tumor, but the tumor pushes the brain as it enlarges. The slowly enlarging tumor protrudes from the internal auditory canal into an area behind the temporal bone called the cerebellopontine angle. Since the balance portion of the eighth nerve is where the tumor arises, unsteadiness and balance problems may occur during the growth of the neuroma. The most common presentation is unilateral hearing loss. The presence of episodes of vertigo along with fluctuating hearing loss is consistent with the diagnosis of Meniere's disease, which is also characterized by tinnitus or a ringing sound in the ear. Some patients also experience a pressure sensation in the ear. Episodes occur at regular intervals for years and may also be marked by periods of remission. The cause is an increase in the volume of endolymph. Vestibular neuronitis presents as acute onset of vertigo, nausea, and vomiting lasting for several days. As this condition is not clearly inflammatory in nature, neurologists often refer to it as vestibular neuropathy. The etiology is unknown, but it appears to be a sudden disruption of afferent neuronal input from the left and right inner ears. This imbalance in vestibular neurologic input to the central nervous system causes symptoms of vertigo. Besides the unilateral caloric weakness, electronystagmography reveals a directional preponderance and beating nystagmus away from the affected side. Workup of patients with vestibular neuronitis begins with a thorough history and physical examination. Audiometry and vestibular testing are the cornerstones of diagnosis, with imaging and laboratory studies being guided by findings on examination. Bedside examination includes a doll's eye test, head-shaking nystagmus, dynamic visual acuity, caloric testing, rotational testing, past pointing, Romberg and Fukuda tests, and tandem walking. Patients with this condition do not generally have auditory symptoms. Dix-Hallpike maneuver is negative. Labyrinthitis is an inflammation or dysfunction of the vestibular labyrinth, which is a system of intercommunicating cavities and canals in the inner ear. The syndrome is defined by the acute onset of vertigo that is commonly associated with head or body movement. Nausea, vomiting, and malaise often accompany the vertigo. The pathophysiology of this syndrome is not completely understood, but a dysfunction of the vestibular apparatus is clearly present when labyrinthitis occurs. Many cases of labyrinthitis are associated with systemic or viral-like illnesses. Suppurative or bacterial labyrinthitis is rare, but it should be considered in patients with acute or chronic otitis media. Cases are reported in association with meningitis, but the presentation of meningitis often overwhelms the vestibular symptoms. Dix-Hallpike manuever is negative.

A 65-year-old Caucasian man presents with a 3-day history of severe dizziness. The symptoms are exacerbated by turning his head and relieved by lying still. He reports nausea and vomiting for the first 2 days of his illness but successfully eats breakfast on the day he is seen in the clinic. He denies hearing loss and tinnitus. His past medical and surgical histories are unremarkable. He has no previous exposure to ototoxic drugs and denies further neurologic symptoms. The otologic examination is without abnormality. Weber testing with a 512 Hz tuning fork is to midline. Romberg and Fukuda testing indicate right-sided pathology. Other than a crisp left-beating nystagmus, cranial nerve examination is normal. Vertigo is experienced after the Dix-Hallpike maneuver. Nystagmus is observed after a few seconds of lying down during the maneuver. Question What is the most likely diagnosis? 1 Acoustic neuroma 2 Benign positional vertigo 3 Meniere's disease 4 Vestibular neuronitis 5 Viral labyrinthitis

5 Start tapering the levodopa dose. Explanation Parkinsonism poorly responsive to levodopa suggests a diagnosis other than Parkinson's disease. Some possibilities are certain forms of secondary Parkinsonism (e.g., from atherosclerosis or multiple trauma) and atypical Parkinsonian syndromes (also called Parkinson-plus syndromes), such as corticobasal degeneration, progressive supranuclear palsy, frontotemporal dementia, dementia with Lewy body disease, and multiple system atrophy. This patient's hyperextended posture, backward falls, and ocular signs are strongly suggestive of progressive supranuclear palsy (PSP, or Steele-Richardson-Olszewski syndrome). Alien limb phenomenon, apraxia, and cerebellar signs may also be seen in this condition. The deficits in ocular motility are often perceived as blurred vision by the patient. This condition progresses more quickly than Parkinson's disease and it usually has a fatal outcome in 5-6 years. While the dramatic response to levodopa seen in Parkinson's disease does not occur, some patients improve transiently, and a trial with doses higher than those used for Parkinson's is warranted. If there is little or no response, or intolerable side effects develop (e.g., dystonia, hallucinations, delusions), the dose is slowly tapered (abrupt withdrawal can worsen the motor symptoms) and withdrawn. Small doses of levodopa sometimes produce modest functional improvement. This patient's hallucinations and dystonias are probably caused by the high levodopa/carbidopa doses, and they should improve after tapering the drug. Specific treatment with antipsychotics is therefore not indicated, and the treatment might worsen the patient's already severe movement disorders and postural instability. Amantadine is useful as an adjunct to levodopa in patients with Parkinson's disease, and it is effective at reducing levodopa-induced dyskinesia. However, antiparkinsonian therapy yielded mediocre efficacy and is causing severe side effects in this patient; therefore, it should be discontinued. For the same reasons, anticholinergics such as benztropine are not useful. Syndrome: Corticobasal degeneration Clinical picture: Asymmetric, akinetic-rigid parkinsonism, apraxia, dystonia, sensory disturbances; dysarthria, and dysphagia are late findings. Comments: Poor response to levodopa and other antiparkinsonian drugs. Evolution is faster than in Parkinson's disease. Treatment is supportive. Syndrome: Frontotemporal dementia parkinsonism - chromosome 17 Clinical picture: Behavioral disturbances, occasional psychosis, cognitive impairment; motor signs (bradykinesia, rigidity) develop later. Comments: Poor response to levodopa and other antiparkinsonian drugs. Evolution is faster than in Parkinson's disease. Treatment is supportive. Syndrome:Progressive supranuclear palsy Clinical picture: Parkinsonism, hyperextended posture, supranuclear gaze palsy, dysphagia, and dysarthria. Cognitive deficits in advanced disease. Comments: Poor response to levodopa and other antiparkinsonian drugs. Evolution is faster than in Parkinson's disease. Treatment is supportive. Syndrome:Multiple systems atrophy Clinical picture: Cerebellar signs, autonomic, and parkinsonian features. Includes patients formerly diagnosed as olivopontocerebellar atrophy, striatonigral degeneration, and Shy-Drager syndrome. Comments: Patients with dysautonomia and postural hypotension may benefit from fludrocortisone or midodrine

A 67-year-old man is brought in with a 4-month history of tremors, blurred vision, and repeated falls. AN MRI performed at that time showed diffuse cerebral and brainstem atrophy. Two months ago, another healthcare provider prescribed levodopa in progressively higher doses; it afforded only marginal, transient improvement. Two weeks ago, he began to suffer visual hallucinations, confusion, occasional contractures in his neck, and choreoathetoid movements in his upper limbs. His past medical history includes essential hypertension that is well controlled with hydrochlorothiazide and an uneventful cholecystectomy 30 years ago. The patient is a retired accounting clerk and he led a fairly active life before the symptoms began. Currently, he is using near-maximal doses of levodopa/carbidopa. Vital signs are within normal range. On physical examination, he is confused and disoriented to time and place. There is a large bruise on his back from when he fell backward over the living room table. His posture is rigid, with hyperextension of the trunk and a low-amplitude, high-frequency tremor in both hands, as well as a coarser tremor in his right arm. He has difficulty following the examiner's finger with his eyes, particularly during horizontal deviation. His gait is unstable; he sometimes has to support himself against the walls to avoid falling. The remainder of the neurological and physical examinations shows no abnormalities. Question What is the most appropriate next step? 1 Raise the levodopa dose. 2 Add benztropine. 3 Start haloperidol. 4 Start amantadine. 5 Start tapering the levodopa dose.

2 Long head of the biceps brachii tendon rupture The long head of the biceps brachii passes through the bicipital groove of the humeral head and attaches to the superior rim of the glenoid. It functions to suppress the humeral head on the glenoid surface, but it is exposed to friction rubbing over time. Therefore, it is subject to rupture, even with what seems like minimal trauma. The distal biceps rupture may occur, but it is less common, more painful, and requires much more force to rupture. The short head of the biceps brachii attaches to the coracoid process of the scapula and is less likely to rupture since there is minimal irritation to this tendon from use. The supraspinatus is a rotator cuff muscle and would not cause a bulge when it ruptures. The triceps brachii muscle is on the posterior arm and would not cause bulge in the anterior arm if it ruptured.

A 67-year-old man presents with the complaint of a "tender bulge in his right dominant arm" following an attempt at lifting his couch 2 days ago. He admits to having mild chronic shoulder pain and taking ibuprofen (Motrin) for relief. While attempting to lift the couch, he felt a snap in his shoulder and noticed a bulging in his anterior arm. Question What anatomic structure is most likely injured? 1 Distal biceps brachii tendon rupture 2 Long head of the biceps brachii tendon rupture 3 Short head of the biceps brachii tendon rupture 4 Supraspinatus tendon rupture 5 Triceps brachii tendon rupture

4 Postinfectious glomerulonephritis Explanation Of the listed choices, this pediatric patient most likely has postinfectious glomerulonephritis (PIGN) or poststreptococcal glomerulonephritis. The diagnosis is supported by both her history and the test results. 5-10% of patients with streptococcal pharyngitis may develop this condition. Though the test is not specific, the antistreptolysin O, along with the history of pharyngitis, help establish a recent streptococcal infection for this patient. PIGN falls under the broader category of nephritic syndromes characterized by hematuria and RBC casts, with possible proteinuria, edema, hypertension, and elevated serum creatinine. Treatment for PIGN is supportive, and normal renal function returns for the vast majority of patients. While the hematuria and presence of leukocyte esterase are consistent with acute cystitis, the rest of the history and findings are not. The classic presentation of acute cystitis includes dysuria and frequency. The remainder of the patient's presentation (edema, elevated BP, RBC casts, and serum tests) is inconsistent with acute cystitis. The leukocyte esterase is not specific for infection; in this case, it indicates inflammation in the urinary tract. Analgesic abuse nephropathy is a chronic tubulointerstitial nephritis caused by cumulative lifetime use of large amounts (i.e., ≥ 2 kg) of certain analgesics. Though this patient was dosed with acetaminophen, her use was in small amounts and only for a few days. The history can rule out this diagnosis. Diabetic nephropathy is an important diagnosis to keep in mind when evaluating renal dysfunction, as it is the most common cause of nephrotic syndrome. The diagnosis includes a history of diabetes and demonstrated proteinuria, which can be confirmed by various methods. This patient has no history of diabetes, and even if she presented with elevated glucose, diabetic nephropathy would not be expected to develop for many years. Renal cell carcinoma (RCC) is suggested by persistent hematuria, especially in individuals over age 50 or those at high risk. The serum findings in this patient are not consistent with the diagnosis of RCC. CT, MRI, and sometimes renal biopsy help confirm the diagnosis of RCC.

A 7-year-old girl presents with a 1-day history of bloody urine. The grossly bloody urine scared both the girl and her parents, but she denies dysuria and frequency. No trauma or sexual abuse has occurred. The parents deny recent fever in the patient, but they note that she had a fever for a few days accompanying a sore throat. She was given acetaminophen at an appropriate dosage for her weight, and about 3 days of some leftover amoxicillin; both the fever and pharyngitis then resolved. Her past medical history is unremarkable for any chronic illnesses. Her only medication is a multivitamin, and she has NKDA. She has had no surgeries, and family history is unremarkable for urinary tract disorders or any bleeding disorders. On physical exam, she appears interactive and in no apparent distress. She is well-nourished, non-obese, and perhaps mildly edematous. Vitals are a temperature of 99.0°F and a BP of 138/85 mm Hg; pulse is 98, and respiratory rate is 20. No rashes are found. Cardiac exam reveals normal rate and rhythm; there are no murmurs or rubs. On abdominal exam, her abdomen is non-distended, non-tender, and without masses or hepatosplenomegaly. She has no CVA tenderness. A urinalysis is performed; the significant findings are as follows: Protein 2+ Glucose Negative Ketones Negative Blood 4+ Nitrites Negative Leukocyte esterase 1+ Microscopic analysis is performed, confirming the presence of red blood cells (RBCs) that are dysmorphic and indicate RBC casts. Blood tests are ordered, revealing a complete blood count within normal limits and a complete metabolic panel with elevated creatinine. The patient's antistreptolysin O level is elevated, and her serum complement level is low. Question What is the most likely diagnosis for this patient? 1 Acute cystitis 2 Analgesic abuse nephropathy 3 Diabetic nephropathy 4 Postinfectious glomerulonephritis 5 Renal cell carcinoma

4 Pseudomonas aeruginosa Explanation Ear pain and drainage in an elderly diabetic patient must raise concern about malignant external otitis. The swelling and inflammation of the external auditory meatus strongly suggest this diagnosis. This infection usually occurs in older patients with poorly controlled diabetes, and it is almost always caused by P. aeruginosa. It can invade contiguous structures, including the facial nerve or temporal bone; it can even progress to meningitis. S. pneumoniae, H. influenzae, and M. catarrhalis frequently cause otitis media, but not external otitis. Candida albicans almost never affects the external ear.

A 70-year-old woman with long-standing type 2 diabetes mellitus presents due to pain in the left ear along with purulent drainage. On physical examination, the patient is afebrile. The pinna of the left ear is tender, and the external auditory canal is swollen and edematous. The white blood cell count is normal. Question What organism is most likely to grow from the purulent drainage? 1 Candida albicans 2 Haemophilus influenza 3 Moraxella catarrhalis 4 Pseudomonas aeruginosa 5 Streptococcus pneumoniae

2 Asbestosis Explanation The correct answer is asbestosis. Asbestosis is an interstitial fibrosis that occurs from lung irritation caused by exposure to asbestos; it is often seen in patients who have an extensive work history of shipbuilding and construction. Pleural calcifications or plaques are typically seen with this disease. Treatment for this disease is supportive. Silicosis, siderosis, byssinosis, and tuberculosis do not cause the calcified pleural plaques that are a typical presentation for asbestosis. Silicosis is often found in patients with occupations such as rock mining or sandblasting. Siderosis is often found in patients with occupations in mining and welding. Byssinosis is found in patients who work in textiles.

A 72-year-old man presents with progressive shortness of breath over the years. He denies chest pain or a history of smoking. The patient was in the construction business for many years, and before that he worked as a shipbuilder. Chest X-ray reveals marked interstitial fibrosis and calcified pleural plaques on the lateral chest wall. Question What is the most likely diagnosis? 1 Silicosis 2 Asbestosis 3 Siderosis 4 Byssinosis 5 Tuberculosis

4 Refer for physical therapy Explanation Rotator cuff tears (RCT) are a common clinical issue in geriatric patients, and there has been a long debate on how to approach restoring shoulder function in these patients. Treatment options usually consist of non-surgical and surgical options. Many times the majority of patients are initially placed on a trial of conservative therapy, which would include physical therapy. Refer this elderly patient with a rotator cuff tear to physical therapy to attempt regaining range of motion by stretching and strengthening the remaining rotator cuff. If the patient is younger or an active athlete and experiences a sudden rotator cuff tear, they should be referred to an orthopedic surgeon for immediate repair. There is little chance this patient will need repair. CT of the shoulder would not be an appropriate test to order at this time. It may show the tear, but little clinical information will be gained unless he becomes a candidate for surgery. Injection of a steroid may improve his painful symptoms, but it also increases the risk of worsening the tear by weakening the tendons. Do NOT place this elderly patient in a shoulder immobilizer, as doing so will decrease the chance of obtaining the return of function by making the shoulder joint stiff.

A 73-year-old man presents with the inability to actively raise his left non-dominant arm to retrieve plates from the kitchen cabinet. He further describes the inability to retrieve any objects with his left hand/arm because of the limited range of motion. He is worried because he is the only driver in his household, and he does not want to lose his driving privileges. He has a longstanding history of chronic shoulder impingement syndrome. On exam, he is found to have a (+) positive drop arm test. Question What initial treatment is recommended for the most likely diagnosis? 1 Immediate subacromial steroid injection 2 Immediate surgical repair 3 Order CT of the left shoulder 4 Refer for physical therapy 5 Place in shoulder immobilizer

2 Proximal anteromedial tibia Explanation Insertion of an intraosseous needle directly into the anterior tibia is relatively easy and will ensure rapid vascular access. This infant's clinical presentation of decreased activity, mottled and dusky skin, and weak pulses suggests severe volume depletion. Whatever the cause—in this emergent situation that can be assessed later—rapid fluid resuscitation and possibly pressors are indicated. A peripheral scalp vein would be very difficult to cannulate in this volume-depleted infant. Blood would be shunted away from the periphery to more vital organs. Also, these small caliber veins would not allow a large bore needle for rapid fluid resuscitation. The saphenous vein, though larger than scalp veins, would also be difficult to cannulate due to shunting. The distal radius is a site chosen for intraosseous access in the adolescent. In an infant, it is not yet fully developed. The sternum is a site used for intraosseous access in the adult and is part of a system called First Access for Shock and Trauma (FAST) and works by a hand driven, push-pull mechanism.

An anxious-looking young mother brings a 6-week-old female infant to your office wrapped in many blankets. When your nurse unwraps her, she appears lethargic; she has dusky and mottled skin and very weak pulses. As your staff quickly provides ventilation and compressions, you prepare to establish vascular access. Question What is the best site for immediate vascular access for this infant? 1 Peripheral scalp vein 2 Proximal anteromedial tibia 3 Saphenous vein 4 Distal radius 5 Sternum

5 Administration of crystalloids and obtain type and crossmatch for packed RBCs Explanation This patient probably has a grade 3 placental abruption, characterized by external uterine bleeding and fetal demise, as well as maternal hemodynamic instability. Patients with placental abruption should have adequate intravenous access established. Crystalloid fluids are initially used to resuscitate the patient. Blood for transfusion should be typed and crossed (usually at least 4 units). Coagulation studies should be drawn to assess for signs of coagulopathy. Fibrinogen is usually given if the fibrinogen level is <100 mg/dL. Platelets may be administered if the platelet count is <50,000 /uL. In the case of fetal demise, vaginal delivery is generally preferred over cesarean section. Placental abruption (abruptio placenta) is defined as the premature separation from the uterus of a normally implanted placenta. As the placenta separates, there is a large amount of bleeding which irritates the uterus and causes uterine contractions, as well as fetal distress because fetal perfusion is compromised. The incidence of placental abruption ranges from 1 in 75 to 1 in 225 births. Placental abruption is classified by degree of separation. Grade 1 abruption presents with slight vaginal bleeding, mild uterine irritability, and a normal fetal heart rate tracing. Maternal blood pressure is normal, as is the maternal serum fibrinogen level. A grade 2 abruption is characterized by mild-to-moderate vaginal bleeding and pronounced uterine irritability, which may include tetanic uterine contractions. The patient may exhibit orthostatic blood pressure changes and often has an elevated pulse. The fetal heart rate usually shows some evidence of distress. Maternal fibrinogen levels are usually reduced to 150 to 250 mg/dL. A patient with a grade 3 abruption will have moderate-to-severe vaginal bleeding (unless concealed in the uterus), painful, tetanic uterine contractions, and hypotension. The fetus will be dead, and the patient is likely to exhibit a coagulopathy with fibrinogen levels less than 150 mg/dL, thrombocytopenia, and reduced levels of clotting factors. Abruptio placenta is associated with several risk factors. Maternal hypertension is one of the most commonly associated risk factors identified in patients who develop placental abruption, especially those with grade 3 abruption (where 40-50% of patients will have hypertensive disease of pregnancy). A history of prior abruption, tobacco abuse, cocaine abuse, poor nutrition, and chorioamnionitis are also associated with an increased risk for abruption. Uterine trauma is also associated with a risk for placental abruption, causing 1-2% of grade 3 abruptions. Patients with trauma in pregnancy may present with minimal physical evidence of trauma, but they will still have a significant abruption that can progress from grade 1 to grade 3 within 24 hours. Multiple gestations and polyhydramnios can also cause placental abruption if the uterus decompresses rapidly during labor. Placental abruption will classically present with painful vaginal bleeding in the third trimester of pregnancy (80% of patients present with bleeding), though the amount of bleeding may be concealed within the uterus (20%). Ultrasound is used to rule out placenta previa (the other common and dangerous cause of third trimester bleeding) and may identify placental abruption. Unfortunately, the ultrasound appearance of an abruption may lag behind the clinical degree of bleeding, so this imaging modality cannot rule out abruption. The prognosis and management of abruptio placenta depend greatly on the gestational age of the fetus, as well as the grade of the abruption. A grade 1 abruption with a term fetus can often be managed with a controlled induced delivery; since blood is a strong uterine irritant, many of these patients will deliver relatively quickly. The management of preterm pregnancies, however, is less clear and depends on the degree of abruption and fetal distress, with the risks of premature delivery measured against the risks of progression of the abruption.

Case A 17-year-old G1, P0 girl presents after being found in a crack house by local police. She was initially cooperative, but she is now experiencing severe abdominal pain, and she has developed vaginal bleeding. She says that she is pregnant, but she has not received any prenatal care. Examination reveals a blood pressure of 90/50 mm Hg, pulse of 120/min, and respiratory rate of 25/min. She is diaphoretic and clammy. Her uterus measures 25 cm from the pubic symphysis, but no fetal heart tones can be appreciated. There is a large amount of dark blood around the vagina. Ultrasound reveals an intrauterine fetal demise and a hyperechoic retroplacental hematoma. Labs are pending. Question What is the next best step in the management of this patient? 1 Emergent labor by artificial rupture of membranes 2 Emergent labor induction with Pitocin 3 Administration of fibrinogen 4 Emergency cesarean section 5 Administration of crystalloids and obtain type and crossmatch for packed RBCs

5 Pneumothorax Explanation This patient is presenting with a spontaneous primary pneumothorax. A finding of a pleural line on chest X-ray (CXR) is diagnostic for this condition. A pneumothorax is a condition in which air is introduced into the lung cavity, either spontaneously or by trauma. In young people without a known pulmonary pathology (more common in smokers especially), a pneumothorax may occur spontaneously. In older patients, other causes, such as a lung tumor or severe pulmonary disease, can lead to the collapse of the lung. As the lung collapses, the line from the edge of the pleura may be visible on CXR. Bronchiectasis is a chronic condition of the large bronchi associated with coughing, dyspnea, and wheezing. Cystic fibrosis is a common cause of bronchiectasis. This patient does not have a cough, wheezing, or a history of chronic pulmonary disease. Bronchitis is a rather vague term referring to acute or chronic inflammation of the bronchi. A viral infection is a common cause of acute bronchitis. Chronic obstructive pulmonary disease (COPD) is a common cause of chronic bronchitis. This patient's history and physical do not support any bronchial inflammation; they are not suggestive of an acute infection. A foreign body in the bronchus can cause acute shortness of breath, chest pain, and some of this patient's physical findings. Typically, an adult patient should be able to provide a history that no foreign bodies were aspirated; an X-ray would usually identify a foreign body. Furthermore, a foreign body would not produce the pleural line unless it caused a pneumothorax, so an airway foreign body is unlikely in this patient. Pneumonia can cause chest pain and shortness of breath, but the patient would likely feel worse; there would be a history of malaise, fatigue, fever, chills, cough, and possibly nausea. The patient's X-ray and physical exam findings are not found with pneumonia. If pneumonia causes percussive changes on the lung exam, it produces dullness to percussion, not hyperresonance.

Case A 22-year-old man presents with a sudden onset of shortness of breath and right-sided chest pain. Symptoms began yesterday, and he felt well prior to the onset of symptoms. He denies fever, hemoptysis, and upper respiratory symptoms. He is a 1 pack-per-day smoker; otherwise, he has a noncontributory past medical history. On physical exam, the patient is in mild respiratory distress, with a slightly elevated heart rate and respiratory rate. He is normotensive. His trachea appears deviated to the left. On pulmonary exam, breath sounds are diminished on the right. Hyperresonance is noted on percussion of the right chest compared to the left. Other than tachycardia, his cardiovascular exam is normal. A chest X-ray is obtained, and a pleural line is visible. Question What is the most likely diagnosis? 1 Bronchiectasis 2 Bronchitis 3 Foreign body in the bronchus 4 Pneumonia 5 Pneumothorax

4 No cortical disruption or necrosis, but multiple small isolated hemorrhages throughout the cortex with edema Explanation The correct response is no cortical disruption or necrosis, but multiple, small, isolated hemorrhages throughout the cortex with edema. A cerebral contusion is a focal lesion of the brain, secondary to trauma. Contusions usually occur over the crest of gyri, largest at the surface of the brain and tapering in size into the white matter. Cerebral contusions can occur under the site of impact, or on the contralateral side of the brain (a contrecoup lesion). The CT scan of a cerebral contusion characteristically reveals punctate hemorrhages and edema under the site of impact. A cerebral contusion is defined as "bruising" of the brain without interruption of the cortex. Frequently, a subarachnoid hemorrhage overlies the area of contusion. A biconvex-shaped hematoma over the cerebral hemisphere is indicative of epidural hematoma. A peripheral ring of enhancement around areas of a hematoma is seen 1 to 6 weeks after a traumatic intracerebral hematoma, not in cerebral contusion. Panventricular enlargement is a feature of hydrocephalus.

Case A 23-year-old man presents after being thrown from a motorcycle onto a cement median. He is awake and appears alert during the exam. He believes that his helmet was knocked off at the point of impact, but he cannot recall the specifics of his crash. A head CT scan is consistent with a cerebral contusion. Question What finding on a CT scan would you see with this diagnosis? 1 A peripheral ring of enhancement around areas of hematoma 2 Biconvex shaped hematoma over the cerebral hemisphere 3 No cortical disruption, but tissue necrosis, edema, and punctate hemorrhages 4 No cortical disruption or necrosis, but multiple small isolated hemorrhages throughout the cortex with edema 5 Enlarged lateral ventricles, third and fourth ventricles, and basal cistern; normal or absent sulci

4 Laparoscopy Explanation Laparoscopy is considered the gold standard in diagnosis of endometriosis. Laparoscopy allows visualization of the islands of abnormally implanted endometrial tissue, but visual inspection alone has a high false positive rate. Confirmation requires biopsy of the abnormal areas, with histopathology demonstrating endometrial epithelium, glands, stroma, or hemosiderin-laden macrophages. A visualization of the uterine cavity and tubes by hysterosalpingography is useful in delineating anatomic defects, such as a bicornuate uterus, as well as mechanical obstruction by adhesion or stenosis as seen post pelvic inflammatory disease. It is not of any use in endometriosis. FSH and LH levels may be used in the workup of infertility, but they provide no information on endometriosis. Ultrasonography is incorrect. In general, imaging techniques are not particularly helpful in diagnosis. It is useful in ruling out other pelvic pathology. Pap smear with endocervical curettage is mostly a screening procedure for cervical and endocervical pathology. It can help in the identification of endometrial dysplasia and malignancy, but not endometriosis.

Case A 23-year-old woman presents with a history of chronic pelvic pain. She attained menarche at 13, and she has had regular periods. She has been experiencing severe pain during menses; the pain has been increasing in frequency. She has never had intermenstrual bleeding, and she regularly uses barrier contraception. She has refrained from intercourse for over 6 months, however, due to the pain it causes her. Her vital signs and general physical examination appear to be normal. A vaginal exam reveals cervical motion tenderness with reduced mobility and nodularity of the uterosacral ligaments. Question For a definitive diagnosis, what is the diagnostic test of choice? 1 HSG 2 FSH and LH levels 3 Ultrasonography 4 Laparoscopy 5 Pap smear with endocervical curettage

2 Increased production of allergy-specific IgG Explanation The correct response is increased production of allergy-specific IgG. Allergy immunotherapy, more commonly known as allergy shots, are given to individuals who have an allergic reaction to common allergens (e.g., mold or pollen from grasses, ragweed, trees). A small amount of the allergen is injected into the patient and the body starts making antibodies to the allergen; this allows the body to fight the allergen and relieves the symptoms of the allergic reaction. The body's main response is increased production of IgE-blocking antibodies and allergy-specific IgG. They also increase IL-10 production, which has anti-inflammatory effects. Allergy shots do not decrease IgE or IgA production. They also do not degrade allergy-specific antibodies.

Case A 25-year-old woman presents with watery eyes and a runny nose. The symptoms get worse after she has been outside, especially if she plays in the grass with her 2-year-old. Allergy testing reveals that she is highly allergic to several grass pollens. Allergy shots using grass pollens as the antigens might be helpful. Question Considering the most likely diagnosis and ultimate treatment intervention, what mechanism will reduce the patient's symptoms? 1 Reduced levels of IL-10 2 Increased production of allergy-specific IgG 3 Degradation of allergy-specific antibodies 4 Decreased production of allergy-specific IgE 5 Decreased production of allergy-specific IgA

4 Eliminate 4 units of NPH insulin before dinner. Explanation The correct answer is to eliminate 4 units of NPH insulin before dinner. The first step in managing insulin therapy is to eliminate low blood sugars. This patient is experiencing nocturnal hypoglycemia, which stimulates the release of counterregulatory hormones to increase the blood sugar, resulting in rebound hyperglycemia. This would account for his elevated fasting blood sugar readings. The NPH insulin has an onset of action of 1-2 hours and a peak effect in 6-12 hours, so decreasing the dinner dose 10-20% will decrease the likelihood of 3 AM hypoglycemia and the subsequent morning rebound hyperglycemia. Eliminating 2 units of regular insulin before dinner may have some effect on the patient's 3 PM hypoglycemia but at the expense of higher 11 PM blood sugar readings. The regular insulin has an onset of action in 30-60 minutes, with a peak effect at 2-4 hours. The current dinner dose of 10 units of regular insulin is providing rapid-acting coverage for that meal with an acceptable average blood sugar of 140 mg/dL at 11 PM. Eliminating 2 units before dinner will result in higher blood sugars at 11 PM and suboptimal control. Adding 4 units of regular insulin before breakfast will serve to lower the elevated fasting blood sugars, but that will do nothing to correct the nocturnal hypoglycemia, which is the main problem. Adding this extra rapid-acting insulin is a reactive response rather than a proactive step to control blood sugars. Adding 2 units of NPH insulin before breakfast will have minimal effect on correcting the elevated morning blood sugars or the nocturnal hypoglycemia. Adding 2 units of NPH insulin before dinner will make the nocturnal hypoglycemia worse.

Case A 35-year-old man with type 1 diabetes presents for an acute visit with uncontrolled blood sugars. He has had high and low readings throughout the day and night. When he called to make this appointment, he was advised to bring in his blood sugar log and check a few 3 AM blood sugars. He is currently on 40 units of NPH and 20 units of regular insulin before breakfast and 20 units of NPH and 10 units of regular insulin before dinner. Time : Average blood glucose mg/dL 7 AM : 393 11 AM : 210 5 PM : 175 11 PM : 140 3 AM : 50 Question What is the most appropriate insulin change based on his average blood glucose readings? 1 Add 4 units of regular insulin before breakfast. 2 Add 2 units of NPH insulin before breakfast. 3 Add 2 units of NPH insulin before dinner. 4 Eliminate 4 units of NPH insulin before dinner. 5 Eliminate 2 units of regular insulin before dinner.

1 Thyroid-stimulating hormone Explanation The answer is thyroid stimulating hormone (TSH), as Graves disease is a type of hyperthyroidism. Lab results expected with hyperthyroidism are as follows: TSH would be lower than normal Total thyroid hormone, free T3, and free T4 would be elevated above normal. Thyroglobulin and thyroid peroxidase are antibodies that are produced in Graves disease, so they would not be low. Graves disease is caused by a generalized overactivity of the entire thyroid gland (hyperthyroidism). The patient appears hot and flushed, and the thyroid gland enlarged. Graves disease is an autoimmune disorder. Antibodies are produced against certain proteins on the surface of thyroid cells, stimulating those cells to overproduce thyroid hormones. In this condition, antibodies are produced against the thyrotropin receptor, thyroglobulin, thyroid peroxidase, and sodium-iodide symporter. The circulating autoantibodies continuously stimulate the thyroid gland via the thyrotropin receptor. The onset of the disease is gradual, and the symptoms may be mistaken for nervousness due to a stressful life situation. Weight loss occurs, and it is followed by other symptoms, such as trembling, muscle weakness of the upper arms and thighs, and insomnia. The pituitary gland releases TSH utilizing a negative feedback loop. In other words, the higher the level of circulating total thyroid hormone, free T3, and free T4, the less TSH is released. In contrast, more TSH is released in response to insufficient levels of thyroid hormone. Communication between the pituitary gland and the thyroid gland through TSH levels controls the levels of thyroid hormone in the blood. Thus, the measurement of TSH in the blood is taken as a measure of thyroid function. The suppression of TSH is an early and highly sensitive marker of thyrotoxicosis in conditions such as Graves disease.

Case A 40-year-old woman presents with anxiety, difficulty sleeping, rapid heartbeat, and tremor in her hands. You note the presence of bulging eyes and suspect Graves disease. Question What lab finding is consistent with the diagnosis when results return lower than normal? 1 Thyroid-stimulating hormone 2 Total thyroid hormone 3 Free T3 and free T4 4 Thyroglobulin 5 Thyroid peroxidase antibody

2 Cytomegalovirus Explanation The correct answer choice is cytomegalovirus. This HIV-positive patient, who is non-compliant with his medications, presents with blurry vision, loss of peripheral vision, as well as spots in his vision. This presentation is common in CMV retinitis. This disease occurs typically in the late stages, particularly when the CD4 count is less than 50 cells/mm3. Fundoscopic examination will reveal areas of infarction and hemorrhage. Classic cotton wool spots may also be noted. Mycobacterium-avium intracellulare typically presents with a variety of symptoms, such as persistent fever, night sweats, weight loss, chronic diarrhea, abdominal pain, and severe anemia. Toxoplasma is a common cause of ring-enhancing masses on CT in patients with HIV. Epstein-Barr is more commonly associated with infectious mononucleosis, nasopharyngeal carcinoma, and Burkitt's lymphoma. HSV-1, or oral herpes, is the virus that causes cold sores. They are tingling or painful spots on the edge of the lip where it meets the skin of the face. HSV-2, or genital herpes, is the cause of very painful genital or anal ulcers. HSV infection of the eye, which causes a keratoconjunctivitis, manifests with an acute onset of pain, watery discharge, itching, blurred vision, lid swelling, and conjunctival injection. Uveitis may also occur with HSV infection; retinitis is rare.

Case A 42-year-old man with a history of HIV presents with blurred vision. The patient also notes occasional small spots in his vision. The patient is non-compliant with his medications. Physical examination reveals vital signs of temperature 100.8°F, BP 114/80 mm Hg, RR 12/min, and pulse 76 BPM. Decreased visual acuity, as well as loss of peripheral vision, is demonstrated. Blood work is notable for a CD4 count of 30 cells/mm3. Question What organism is most likely responsible for this patient's symptoms? 1 Mycobacterium-avium intracellulare 2 Cytomegalovirus 3 Toxoplasma gondii 4 Epstein-Barr virus 5 Herpes simplex

1 Oral amoxicillin Explanation The correct response is oral amoxicillin. The diagnosis in this case is acute otitis media (inflammation of the middle ear), which is a common childhood infection. Infants and children are at highest risk for otitis media; incidence rates are 15-20%, with peaks occurring from 6-36 months and 4-6 years of age. Children who develop otitis media in the first year of life have an increased risk of recurrent acute infection or chronic disease. In the usual course, a child suffering an upper respiratory infection for several days suddenly develops otalgia, fever, and hearing loss. The characteristic features include a bulging, opaque, erythematous tympanic membrane with impaired mobility. Purulent otorrhea may be present, but earache and fever are not always present. Any child with a "fever of undetermined origin" must also be evaluated for a middle ear infection. Bacteria are the primary agents of otitis media. The most common causes in all age groups are Streptococcus pneumoniae (25-40% of cases), followed by Haemophilus influenzae (15-25% of cases). Gram-negative bacilli cause about 20% of otitis media in neonates, but these bacteria are rarely found in older children with otitis media. Less common causes include group A Streptococci and Branhamella catarrhalis. Staphylococcus is a less common cause of chronic otitis media. Normally, children will improve clinically within 48 hours after antimicrobial therapy. If there is no improvement, the possibility of a resistant organism must be suspected; trimethoprim-sulfamethoxazole or erythromycin and sulfonamides may be given. The antibiotic of choice is amoxicillin orally; it is effective for both S. pneumoniae and H. influenzae. There is no added advantage of intramuscular injection over oral amoxicillin. An increasing percentage of H. influenzae and Moraxella catarrhalis strains have now become beta-lactamase producing and therefore ampicillin-resistant. Some resistant cases may benefit from a change of antibiotics to erythromycin or sulfonamides. Needle aspiration of the middle ear is only rarely necessary, as in the case of a critically ill child or a child who fails to respond to standard antimicrobial therapy.

Case A 5-year-old boy presents with a sudden high fever and severe pain in the right ear. He has been very irritable since the initial onset of symptoms. He also seems to have some difficulty in hearing. The child has been suffering from a cough and cold for the last 2 weeks. He recently returned to the United States after visiting family abroad with his mother, where they spent time with an ill relative. The mother notes that he exhibited no irritability or signs of ear pain during the plane ride home. Question What is the best step in management of the boy's condition? 1 Oral amoxicillin 2 Intramuscular amoxicillin 3 Intramuscular ampicillin 4 Oral ampicillin 5 Needle aspiration

3 Symptoms can significantly improve with alcohol cessation. Explanation This patient's clinical presentation and diagnostic workup are consistent with dilated cardiomyopathy, and the likely cause is his excessive alcohol intake, rather than genetics. There is a significant amount of risk associated with this disease that cannot be ignored, but he can experience a significant improvement in symptoms and reversal of damage within 3-6 months of abstinence from alcohol. It is appropriate to wait for that 6-month period of time before reassessing for the need of an implantable defibrillator, as his disease is not considered severe at this point and the condition may improve to the point where a defibrillator is no longer necessary. His mild mitral regurgitation is the result of annular dilation related to the disease, not the cause of the disease.

Case A 54-year-old man presents with a 6-month history of increasing intolerance to exercise. He describes "breathlessness" with exertion, as well as fatigue and 2-pillow orthopnea. He denies tobacco use but does admit to 4 or 5 whiskey sours daily for the last 20 years. He is a businessman and often entertains clients, which "involves drinking alcohol." Chest X-ray reveals an enlarged cardiac silhouette. EKG reveals normal sinus rhythm. A surface echocardiogram reveals an ejection fraction of 35%, mild mitral regurgitation, and dilated left ventricle. Question What statement about this patient's illness is true and should be shared with the patient? 1 His cardiomyopathy is likely familial. 2 There is minimal risk with this disease, so he can be treated symptomatically. 3 Symptoms can significantly improve with alcohol cessation. 4 He is in need of an implantable defibrillator emergently. 5 His cardiomyopathy is the result of his heart murmur.

4 Acute posterior myocardial infarction Explanation The correct answer is acute posterior myocardial infarction, as the ECG findings are consistent with this diagnosis and troponin is elevated. ST depression and larger R waves in V1, V2, V3 are consistent with acute posterior myocardial infarction. T wave inversion without significant Q waves would suggest ischemia without myocardial infarction (MI). Also ischemia would not cause an elevated troponin. ST segment elevation and wide Q waves in leads I and AVL would suggest a lateral MI. ST segment elevation and wide Q waves in leads II, III, and AVF would suggest an inferior MI. ST segment elevation and wide Q waves in the anterior leads (V1, V2, V3, and/or V4) would suggest an anterior myocardial infarction.

Case A 63-year-old woman presents with a 1-hour history of left shoulder pain and nausea. She has a past medical history of coronary artery disease and had a stent placed 5 years ago. Troponin is elevated. An ECG shows large R waves and ST segment depression in leads V1, V2, and V3. Question These ECG findings are most consistent with what condition? 1 Acute ischemia without myocardial infarction 2 Acute lateral myocardial infarction 3 Acute inferior myocardial infarction 4 Acute posterior myocardial infarction 5 Acute anterior myocardial infarction

1 Small cell carcinoma Explanation Small cell carcinoma is most frequently a central lesion not associated with cavitation. A hilar mass is associated with small cell carcinoma. Small cell carcinoma is associated with paraneoplastic syndromes. Cushing's syndrome secondary to ectopic ACTH production can sometimes be seen with small cell carcinoma, as can the syndrome of inappropriate antidiuretic hormone production (SIADH). Pathology reveals small, round and spindle-shaped cells with a high mitotic count that reflects the rapid proliferation associated with this tumor type. An adenocarcinoma is usually seen peripherally on X-ray. An adenocarcinoma usually produces mucus and often involves the pleural. Adenocarcinoma is the most common lung cancer among people who do not smoke. Adenocarcinomas may develop in areas of the lung where there is scarring or fibrosis. Wilms tumor is a pediatric malignancy of the kidney. It can metastasize to the lungs. Large cell carcinoma is usually seen peripherally on X-ray. Large cell carcinoma shows poorly differentiated cells. It does not show squamous features or glandular features. Large cell carcinoma does not react to mucicarmine staining. Large cell carcinoma is also called undifferentiated carcinoma or anaplastic carcinoma. The cell of origin is unknown. Squamous cell carcinoma is centrally located. Cavitation can also sometimes be seen with X-ray. Keratin formation, keratin pearl formation, and intercellular bridges can be seen with squamous cell carcinoma.

Case A 65-year-old man presents with a 25-pound unexplained weight loss over the last 2 months. He also has noticed a change in his usual cough. He sees his family doctor. Upon questioning for the medical history, the doctor discovers that he has a 60-pack/year history of smoking as well as dyspnea. On physical examination, he appears to have Cushing's syndrome. X-rays show a central lesion with no cavitation. A hilar mass is seen on chest X-ray. CT-guided lung biopsy reveals round cells with a high mitotic count. Question What is the most likely diagnosis? 1 Small cell carcinoma 2 Adenocarcinoma 3 Metastatic Wilms tumor 4 Large cell carcinoma 5 Squamous cell carcinoma

2 Schizophrenia Explanation The most likely diagnosis is schizophrenia. Schizophrenia requires the presence of at least 2 of the following symptoms: delusions hallucinations speech disorganized or catatonic behavior diminished mood or expression of emotion These symptoms must be persistent for at least 6 months and cause a significant impairment in work and social function. A delusional disorder requires the presence of delusions that persist for at least one month. Delusions are defined as fixed beliefs that are not amenable to change in light of conflicting evidence. The patient in this example is exhibiting a persecutory delusion (i.e., belief that one is going to be harmed or harassed by another person or group). In order to make the diagnosis of delusional disorder, the patient must not meet criteria for the diagnosis of schizophrenia. Patients with schizotypal personality disorder are often considered odd and eccentric due to their unusual beliefs and way of dressing, as well as their difficulty relating to people. They are often suspicious of other people. Nevertheless, they may experience anxiety or even be depressed at their inability to form relationships with other people. Delusions and hallucinations are not prominent; therefore, this patient's symptoms are more consistent with the diagnosis of schizophrenia. Substance-induced psychotic disorder is diagnosed when a patient presents with delusions and/or hallucinations and there is a history of substance abuse. The symptoms may occur during intoxication or withdrawal from drug use. There is no history of drug use in this patient and the symptoms have persisted for decades, making this diagnosis unlikely. Psychotic disorder due to a medical condition is diagnosed when a patient exhibits delusions and/or hallucinations as a consequence of a known pathophysiological event or condition, such as a malignancy. There is no history of medical disease in this patient. Furthermore, this is an unlikely diagnosis in a patient whose symptoms have been present for decades.

Case A 70-year-old woman has been refusing to leave her room at the nursing home facility where she resides. She says that people are following her, and she even refuses to go out with her daughter. She has a long history of mental illness; her ex-husband had her committed to a state hospital, which is where she had resided for over 30 years. On interview, it is difficult to obtain a history; her thinking is disordered. When asked why he committed her, she says that she believes her husband was trying to kill her. Question What is the most likely diagnosis? 1 Delusional disorder 2 Schizophrenia 3 Schizotypal personality disorder 4 Substance-induced psychotic disorder 5 Psychotic disorder due to a medical condition

4 First-born male child Explanation Pyloric stenosis is the most common cause of gastric outlet obstruction in infants. Clinical features characteristic of pyloric stenosis include projectile vomiting, weight loss, and dehydration beginning around the third week of life. Diagnostic tests show a pylorus with hypertrophic pyloric musculature. Although the disease is easily treated with surgery, the exact cause of congenital pyloric stenosis is unknown. The etiology is thought to be multifactorial due to various environmental and genetic factors. There is a higher prevalence of pyloric stenosis in boys, with first-born boys having the highest risk. There is a generalized occurrence for siblings of 5-9% and a high concordance rate in monozygotic twins. It is most common in Caucasians and less prevalent in African Americans and Asians. A rare association with developmental delay has also been reported. Suspect environmental factors include infantile hypergastrinemia, myenteric plexus innervation abnormalities, cow's milk protein allergy, and exposure to macrolide antibiotics. Paint exposure has not been shown to have any effect on the development of hypertrophic pyloric stenosis (HPS). Therefore, both genetic and environmental factors are involved in the development of hypertrophic pyloric stenosis (HPS).

Case A mother presents with a 5-week-old male infant for excessive vomiting. He has been having consistent episodes of projectile vomiting 30-40 minutes after feeding and is not gaining weight. The infant has no significant past medical history, although the mother mentions that she began renovating their Victorian-era house while pregnant. He has a 3-year-old sister, who is healthy and has no significant past medical history. Pyloric stenosis is suspected given the infant's projectile vomiting. Question What has congenital pyloric stenosis been most commonly associated with? 1 Female sex of the child 2 Asian origin 3 Penicillin therapy during pregnancy 4 First-born male child 5 Exposure to paint during pregnancy

5 Refer for psychotherapy. Explanation This patient's weight loss is explained by anorexia nervosa, an eating disorder characterized by preoccupation with physical appearance, food restriction, weight loss, and social withdrawal. This patient has lost an alarming amount of weight rapidly, and the anemia, hypoalbuminemia, and mild electrolyte disturbances are common with malnutrition. The best intervention for this patient is a referral for psychotherapy. Appropriate psychotherapy can address the psychiatric illness, and the associated deficiencies should improve with improved food intake. Even though this patient exhibits a very mild hypokalemia, it is not necessary, nor an adequate treatment, to initiate a potassium supplement. The underlying disorder of anorexia nervosa must be addressed, and the potassium should normalize with improved nutrition. This patient exhibits some depressive symptoms, and antidepressant medication may be useful in patients with anorexia nervosa, but it would be inappropriate to prescribe bupropion hydrochloride (Wellbutrin), as decreased appetite and weight loss are common side effects with this medication. Psychotherapy is the mainstay of treatment, and an antidepressant without weight loss potential may be useful. Although approved for AIDS-associated weight loss and anorexia, it would not be appropriate to prescribe dronabinol (Marinol) for this patient, both because it is a non-indicated use and because it would not address the primary problem of her anorexia nervosa. Dronabinol is a cannabinoid, which increases appetite. This patient has a mild hypoalbuminemia, most likely from overall malnutrition and lack of adequate intake. Simply recommending daily protein bars or drinks is not an appropriate intervention; doing so overlooks the patient's inability to consume more food. Treatment of weight loss from anorexia is not to merely telling patients to eat more. Referral to a dietician experienced in eating disorders may be useful for ways to consume more nutrient-dense foods; this would be an adjunct treatment to psychotherapy.

Case An 18-year-old woman presents with weight loss. Her friend has been worried about the patient's weight loss and provides most of the patient's history; the patient remains quiet and reserved. The friend notes that the patient has become very thin over the past several months and her attitude is "listless." The patient's mother died 5 years ago, and the patient admits not feeling comfortable with her father's new wife. She is starting college and admits that she never feels hungry. She is not interested in food; she avoids social situations that involve meals and frequently worries about her appearance. She feels she is not "good enough" for anybody. She denies suicidal thoughts. Physically, she states that she is "okay" and denies symptoms. Her reported weight 7 months ago was 132 lb. She has not had a menstrual period for 4 months, and she has never been sexually active. Her vitals are: Weight 106 lb Height 65" Body mass index 17.6 Pulse 98 Blood pressure 108/72 mm Hg Temperature 97.1°F/36.1°C On physical exam, the patient is thin and somewhat pale. Her affect is flat, and she is soft-spoken. The remainder of her physical exam is normal. Labs results are as follows: Test Result : Normal range : Units Complete blood count (CBC) WBC 6.1 3.6-9.0 K/μL RBC 4.78 4.18-5.22 M/μL Hemoglobin 12.6 12.9-15.5 g/dL Hematocrit 34.5 34.6-50.1 % MCV 79 80.0-100.0 fL MCH 32.2 27.0-34.0 pg MCHC 34.1 30.0-37.0 g/dL RDW 11.7 11.0-17.0 % Platelets 378 140-440 K/μL MPV 9.9 6.5-12.0 fL WBC differential Normal Complete Metabolic Panel (CMP) Sodium 134 134-144 mmol/L Potassium 3.3 3.4-4.9 mmol/L Chloride 100 100-109 mmol/L HCO3 26 20-31 mmol/L Glucose 72 70-99 mg/dL Bun 18 18-Jul mg/dL Creatinine 0.9 0.6-1.2 mg/dL Calcium 9.9 8.8-10.5 mg/dL Albumin 3.4 3.5-5.0 g/dL Total Protein 6.1 6.4-8.2 g/dL AST (SGOT) 21 15-37 U/L ALT (SGPT) 17 May-43 U/L Alk Phosphatase 55 50-136 U/L Total Bilirubin 0.8 0.1-1.2 mg/dL eGFR >60 >60 mL/min/1.73 m2 Human chorionic gonadotropin (hCG) Negative Thyroid-stimulating hormone (TSH) 2.141 0.500-4.700 uIU/mL Question What is the best intervention in this case? 1 Initiate potassium supplement. 2 Prescribe bupropion hydrochloride (Wellbutrin). 3 Prescribe dronabinol (Marinol). 4 Recommend daily protein bars or drinks. 5 Refer for psychotherapy.

2 Copiously irrigate the wound. Explanation This patient's injury is sometimes called a clenched fist injury due to the position of the fist when the injury occurs. This injury may also be called a closed fist injury or "fight bite." Although many occurrences of this type of injury do not initially appear severe, the overall consequences of this injury are very deceiving. The possibility of significant infection is an issue due to there being over 42 different organisms that one may be exposed to from a human bite. Another factor that makes this injury serious is that the injury itself does not often bleed heavily and the underlying injury is hidden by soft tissue. Just range of motion of the injured joint could seal the bacteria that may be introduced during the impact; severe complications can range from cellulitis, osteomyelitis, septic arthritis, or even tenosynovitis. Delay in seeking treatment will only worsen these complications. Treatment should begin promptly and consists of an extreme irrigation of copious amounts of sterile water or an antiseptic solution. Typically this is done with a high-pressure stream of water or solution into the wound. A prompt referral to a hand surgeon would then be appropriate. Incision and drainage of the wound may also be considered, and splinting may occur for stabilization after proper evaluation and cleaning. Bite wounds, however, should never be sutured. Intravenous antibiotics may be administered for the first 24 hours. Once the patient has been discharged, they are usually switched to oral antibiotics for 5-7 days. Antibiotic choice is dependent on those used by the hand surgeons in the community. Not treating the patient as described above would very highly likely lead to devastating consequences with significant morbidity.

Police bring in a 28-year-old male prisoner who was in a fight with another man 2 hours prior to examination. During the fight, your patient struck the other man in the mouth; he sustained a laceration over his first and second knuckle of his right hand. A prompt initial inspection reveals a laceration that is 2.5 cm in length; there is no foreign body and the depths of the wound do not appear to involve bone or tendon. There is noted tenderness of the head of the first metacarpal bone, but no crepitus. There is no limitation to passive or active motion. The patient's last tetanus booster was 1 year ago. His physical examination, aside from his hand, is within normal limits. Question What should be the immediate course of action for this patient? 1 Refer a hand surgeon for a consultation. 2 Copiously irrigate the wound. 3 Discharge him into police custody on oral antibiotics. 4 Initiate intravenous antibiotics as soon as possible. 5 Do nothing else and discharge the patient.

3 Mitral valve prolapse Explanation This patient scenario is highly likely to be mitral valve prolapse (MVP). MVP is surprisingly frequent in otherwise relatively healthy young women (up to 10% of women are diagnosed via echocardiography). MVP is when the mitral valves do not appropriately move as they should, which results in a floppy state or prolapse of the valves. Patients who are asymptomatic and have evidence of a mid-systolic click will typically have a diagnosis of MVP and no sequelae as they age. Those with more severe disease may experience complications, such as rupture of the chordae tendineae, progressive dilatation, or even regurgitation of blood flow. This increases the need for valve replacement as age increases; if significant regurgitation develops, replacement is necessary for about 2% of those originally diagnosed with MVP. While MVP is a murmur that is usually heard during the systolic stage, a patient who has aortic prolapse will have a significant murmur heard best during the diastolic phase. Aortic valve prolapse (and the subsequent aortic regurgitation) has become much less prevalent because a main cause of the condition is rheumatic fever due to untreated β-hemolytic streptococcal infections. Any cardiac valves that have developed stenosis will have a characteristic murmur created from the abnormally narrowed valvular orifice, which is what obstructs blood flow.

Question An otherwise asymptomatic, thin 18-year-old woman presents with a history of scoliosis and the presence of a mid-systolic click that is auscultated during physical examination. The patient most likely has what valvular disease? 1 Mitral stenosis 2 Aortic stenosis 3 Mitral valve prolapse 4 Pulmonic stenosis 5 Aortic valve prolapse

3 Intestinal lymphoma Explanation Although developing cancer secondary to celiac disease is rare, patients with long-standing celiac disease are at highest risk of developing lymphoma. There is some risk of other digestive tract cancers, such as small intestinal adenocarcinoma, esophageal squamous cell, hepatocellular, and colorectal cancers. There is actually a reduced risk of breast cancer. The early stage of celiac disease is characterized by increased accumulation of lymphocytes within lamina propria. Immunologic reaction against a gluten-containing diet is the probable cause of intestinal mucosal damage in patients with celiac disease (immune mechanism). Direct toxic effect of gluten and related peptide (gliadin) is also a possible mechanism of intestinal injury in such patients.

Question A 45-year-old man presents with extreme asthenia and weight loss. He has been suffering from celiac disease for the past 12 years. He is at the highest risk of developing what type of intestinal malignancy? 1 Intestinal leiomyosarcoma 2 Squamous cell anal carcinoma 3 Intestinal lymphoma 4 Carcinoid tumor of intestinal tract 5 Intestinal hemangiosarcoma

3 Percutaneous transluminal angioplasty Explanation The correct choice would be percutaneous transluminal angioplasty because the patient is already on 2 antihypertensives, yet his hypertension is poorly controlled. Also, he is gradually developing bilateral renal artery stenosis. This will probably worsen his hypertension. If there are no contraindications for a surgical repair of the arteriosclerotic artery, it is the preferred course of action. It cures the hypertension without the need for medication. Percutaneous transluminal angioplasty is the procedure of choice in symptomatic stenosis. Additional stenting can also be done. This procedure has shown 90% success rates. It is unliky that adding other antihypertensives, such as diuretics, would be useful. Atorvastatin is used only as adjunctive therapy to lower blood cholesterol in case there is hyperlipidemia. Similarly, a low salt diet to lower BP, cessation of smoking, and low dose aspirin are used as general measures and not as the only line of management. To remove the affected kidney would definitely cure the hypertension, but is too radical. Leaving the patient with only one kidney would be very hard to justify in a case like this.

Question A 52-year-old patient, with a known case of renovascular hypertension, presents with poorly-controlled hypertension. He has been treated with both enalapril and nifedipine. He had been diagnosed with unilateral left renal artery stenosis, but recent tests have demonstrated mild changes in the right renal artery also. What should be the next step in management? 1 Diuretics 2 Reduction of NaCl consumption 3 Percutaneous transluminal angioplasty 4 Add atorvastatin and observe 5 Left nephrectomy


Conjuntos de estudio relacionados

Nouns, Pronouns, Adjectives, and Capitalization

View Set

Ch 45 Pediatric Medication Administration and Calculations

View Set

Украинская литература

View Set

Lippincott for Taylor: Fundamentals of Nursing Chapter 7- Legal Dimensions of Nursing Practice

View Set

PSYC 356 Chp 15 Buss, PSYC 356 Chp 13 MaCrea and Costa

View Set

Physics II Final - MisConceptual Questions

View Set